Download as pdf or txt
Download as pdf or txt
You are on page 1of 52

SFG 2023 |LEVEL 1 |Test 22 |

DO NOT OPEN THIS TEST BOOKLET UNTIL YOU ARE TOLD TO DO SO


T.B.C.: FIAS-SFG23-L1T22 Test Booklet Series
Serial No.

A
TEST BOOKLET
GENERAL STUDIES

Time Allowed: ONE HOUR Maximum Marks: 100


INSTRUCTIONS
1. IMMEDIATELY AFTER THE COMMENCEMENT OF THE EXAMINATION, YOU SHOULD CHECK
THAT THIS TEST BOOKLET DOES NOT HAVE ANY UNPRINTED OR TORN OR MISSING PAGES
OR ITEMS ETC. IF SO, GET IT REPLACED BY A COMPLETE TEST BOOK.
2. Please note that it is the candidate’s responsibility to encode and fill in the Roll Number and Test
Booklet Series A, B, C or D carefully without any omission or discrepancy at the appropriate
places in the OMR Answer Sheet. Any omission/discrepancy will render the Answer Sheet liable
for rejection.
3. You have to enter your Name, Email Id and
Name:
Mobile No. on the test booklet in the Box Email Id:
provided alongside. DO NOT write anything
Mobile No:
else on the Test Booklet.
4. This Test Booklet contains 50 items (questions). Each item is printed in English and Hindi. Each
item comprises four responses (answers). You will select the response which you want to mark
on the Answer Sheet. In case you feel that there is more than one correct response, mark the
response which you consider the best. In any case, choose ONLY ONE response for each item.
5. You have to mark all your responses ONLY on the separate Answer Sheet provided.
See directions in the Answer Sheet.
6. All items carry equal marks.
7. Before you proceed to mark in the Answer Sheet the response to various items in the Test Booklet,
you have to fill in some particulars in the Answer Sheet as per instructions sent to you with your
Admission Certificate.
8. After you have completed filling in all your responses on the Answer Sheet and the examination
has concluded, you should hand over to the Invigilator only the Answer Sheet. You are permitted
to take away with you the Test Booklet.
9. Sheets for rough work are appended in the Test Booklet at the end.
10. Penalty for wrong answers:
THERE WILL BE PENALTY FOR WRONG ANSWERS MARKED BY A CANDIDATE IN THE
OBJECTIVE TYPE QUESTION PAPERS.
(i) There are four alternatives for the answer to every question. For each question for which a
wrong answer has been given by the candidate, one-third of the marks assigned to that
question will be deducted as penalty.
(ii) If a candidate gives more than one answer, it will be treated as wrong answer even if one
of the given answers happens to be correct and there will be same penalty as above to that
question.
(iii) If a question is left bank i.e., no answer is given by the candidate, there will be no penalty
for that question.

Forum Learning Centre: Delhi - 2nd Floor, IAPL House, 19 Pusa Road, Karol Bagh, New Delhi - 110005 | Patna - 2nd floor, AG Palace, E Boring Canal
Road, Patna, Bihar 800001 | Hyderabad - 1st & 2nd Floor, SM Plaza, RTC X Rd, Indira Park Road, Jawahar Nagar, Hyderabad, Telangana 500020
9821711605 | https://academy.forumias.com | admissions@forumias.academy | helpdesk@forumias.academy
Page 1

SFG 2023 |LEVEL 1 |Test 22 |

Q.1) Which of the following led to the Q.4) Consider the following statements
introduction of English Education in India? regarding Wardha Scheme of Basic Education:
1. Charter Act of 1813 1. The detailed national scheme for basic
2. General Committee of Public Instruction, education was formulated by Zakir Hussain
1823 committee.
3. Orientalist and Anglicist Controversy 2. It advocated a free and compulsory
Select the correct answer using the code given nationwide education system through
below: mother tongue for the first seven years of
a) 1 and 2 only schooling.
b) 2 only 3. The scheme could not be put into practice
c) 1 and 3 only due to the start of the First World War.
d) 1, 2 and 3 Which of the statements given above is/are
correct?
Q.2) Consider the following statements with a) 1 only
respect to the Indian National Army (INA) trials: b) 2 only
1. Congress adopted a resolution in the c) 1 and 2 only
Bombay session in support of the Indian d) 1, 2 and 3
National Army’s cause.
2. Prem Kumar Sehgal, Shah Nawaz khan and Q.5) Consider the following statements with
Gurbaksh Singh Dhillon were the first INA reference to the Acropora corals, that has been
prisoners to face the trial. seen recently in news:
3. Following the public protest, British 1. They are the fast-growing corals found in
reduced the death sentence of INA Australia’s Great Barrier Reef.
prisoners to deportation for life. 2. They do not contain zooxanthellae and
Which of the statements given above are derive nourishment from capturing
correct? different forms of plankton.
a) 1 and 2 only 3. They can strongly withstand environmental
b) 2 and 3 only pressures such as rising temperatures,
c) 1 and 3 only cyclones, and pollution.
d) 1, 2 and 3 Which of the statements given above is/are
correct?
a) 1 and 2 only
Q.3) With reference to the Royal Indian Navy
(RIN) revolt, consider the following statements: b) 3 only
1. Only the members of the armed forces c) 1 only
participated in the revolt. d) 1, 2 and 3
2. It demanded withdrawal of Indian troops
from Indonesia. Q.6) With reference to the Sapru Committee of
3. Congress gave its official support to the 1945, consider the following statements:
revolt. 1. The Committee rejected the Muslim
4. Communists and socialist played a major league’s demand for Pakistan.
role in guiding the mutiny. 2. It operated under the mandate of Indian
Which of the statements given above are National Congress.
correct? 3. The Committee recommended the separate
a) 1 and 4 only electorate for Muslims.
b) 2 and 3 only Which of the statements given above is/are
c) 2 and 4 only correct?
d) 2, 3 and 4 only a) 1 and 3 only
b) 1 and 2 only
c) 2 only
d) 1 only

Forum Learning Centre: Delhi - 2nd Floor, IAPL House, 19 Pusa Road, Karol Bagh, New Delhi - 110005 | Patna - 2nd floor, AG Palace, E Boring Canal
Road, Patna, Bihar 800001 | Hyderabad - 1st & 2nd Floor, SM Plaza, RTC X Rd, Indira Park Road, Jawahar Nagar, Hyderabad, Telangana 500020
9821711605 | https://academy.forumias.com | admissions@forumias.academy | helpdesk@forumias.academy
Page 2

SFG 2023 |LEVEL 1 |Test 22 |

Q.1) निम्ननिखित में से निसिे भारत में अंग्रेजी निक्षा िी Q.4) बे नसि निक्षा िी र्धाट योजिा िे सं बंध में
िुरुआत िी? निम्ननिखित िथिों पर नर्चार िरें :
1. 1813 िा चार्ट र एक्ट 1. बे नसि निक्षा िे निए नर्स्तृ त राष्ट्रीय योजिा जानिर
2. सार्ट जनिि निर्दे ि िी सामान्य सनमनत, 1823 हुसै ि सनमनत द्वारा तैयार िी गई थी।
3. प्राच्य और आं ग्ल नर्र्ार्द 2. इसिे स्कूिी निक्षा िे पहिे सात र्र्षों िे निए
िीचे नर्दए गए िूर् िा प्रयोग िर सही उत्तर चुनिए: मातृ भार्षा िे माध्यम से एि मुफ्त और अनिर्ायट
a) िेर्ि 1 और 2 राष्ट्रव्यापी निक्षा प्रणािी िी र्िाित िी।
b) िेर्ि 2 3. प्रथम नर्श्व यु द्ध िुरू होिे िे िारण इस योजिा िो
c) िेर्ि 1 और 3 अमि में िही ं िाया जा सिा।

d) 1, 2 और 3 ऊपर नर्दए गए िथिों में से िौि सा/से सही है /हैं ?


a) िेर्ि 1

Q.2) आज़ार्द नहन्द फौज (INA) िे र्र ायि िे संबंध में b) िेर्ि 2
निम्ननिखित िथिों पर नर्चार िरें : c) िेर्ि 1 और 2
1. आज़ार्द नहन्द फौज िे समथटि में िां ग्रेस िे बं बई सत्र d) 1, 2 और 3
में एि प्रस्तार् अपिाया।
2. प्रेम िुमार सहगि, िाह िर्ाज िाि और गु रबख्श Q.5) हाि ही में समाचारों में र्दे िे गए एक्रोपोरा िोरि िे
नसं ह निल्ों मुिर्दमे िा सामिा िरिे र्ािे पहिे सं र्दभट में निम्ननिखित िथिों पर नर्चार िरें :
आईएिए िैर्दी थे। 1. ये ऑस्ट्रेनिया िे ग्रे र् बै ररयर रीफ में पाए जािे र्ािे
3. जिता िे नर्रोध िे बार्द, अंग्रेजों िे आईएिए िैनर्दयों ते जी से बढ़िे र्ािे िोरि हैं ।
िी मौत िी सजा िो घर्ािर आजीर्ि निर्ाट सि िर 2. उिमें ज़ोक्सें थेिे िही ं होते हैं और प्लैंिर्ि िे नर्नभन्न
नर्दया। रूपों िो पिड़िे से पोर्षण प्राप्त िरते हैं ।
ऊपर नर्दए गए िथिों में से िौि से सही हैं ? 3. र्े बढ़ते तापमाि, चक्रर्ात और प्रर्दूर्षण जैसे
a) िेर्ि 1 और 2 पयाट र्रणीय र्दबार्ों िा दृढ़ता से सामिा िर सिते हैं
b) िेर्ि 2 और 3 ऊपर नर्दए गए िथिों में से िौि सा/से सही है /हैं ?
c) िेर्ि 1 और 3 a) िेर्ि 1 और 2
d) 1, 2 और 3 b) िेर्ि 3
c) िेर्ि 1
Q.3) रॉयि इं नियि िेर्ी (RIN) नर्द्रोह िे सं र्दभट में , d) 1, 2 और 3
निम्ननिखित िथिों पर नर्चार िरें :
1. िेर्ि सिस्त्र बिों िे सर्दस्ों िे नर्द्रोह में भाग निया। Q.6) 1945 िी सप्रू सनमनत िे सं र्दभट में निम्ननिखित
2. इसिे इं िोिेनिया से भारतीय सै नििों िी र्ापसी िी िथिों पर नर्चार िीनजये :
मां ग िी। 1. सनमनत िे मुखिम िीग िी पानिस्ताि िी मां ग िो
3. िां ग्रेस िे नर्द्रोह िो अपिा आनधिाररि समथटि िाररज िर नर्दया।
नर्दया। 2. यह भारतीय राष्ट्रीय िां ग्रेस िे अनधर्दे ि िे तहत
4. साम्यर्ानर्दयों और समाजर्ार्दी िे नर्द्रोह िो निर्दे नित सं चानित होता है ।
िरिे में प्रमुि भू नमिा निभाई। 3. सनमनत िे मुसिमािों िे निए पृथि निर्ाट चि मंिि
ऊपर नर्दए गए िथिों में से िौि से सही हैं ? िी नसफाररि िी।
a) िेर्ि 1 और 4 ऊपर नर्दए गए िथिों में से िौि सा/से सही है /हैं ?
b) िेर्ि 2 और 3 a) िेर्ि 1 और 3
c) िेर्ि 2 और 4 b) िेर्ि 1 और 2
d) िेर्ि 2, 3 और 4 c) िेर्ि 2
d) िेर्ि 1

Forum Learning Centre: Delhi - 2nd Floor, IAPL House, 19 Pusa Road, Karol Bagh, New Delhi - 110005 | Patna - 2nd floor, AG Palace, E Boring Canal
Road, Patna, Bihar 800001 | Hyderabad - 1st & 2nd Floor, SM Plaza, RTC X Rd, Indira Park Road, Jawahar Nagar, Hyderabad, Telangana 500020
9821711605 | https://academy.forumias.com | admissions@forumias.academy | helpdesk@forumias.academy
Page 3

SFG 2023 |LEVEL 1 |Test 22 |

Q.7) With reference to the Indian provincial Which of the statements given above are
elections of 1946, consider the following correct?
statements: a) 1, 2 and 3 only
1. The Congress failed to get majority in any of b) 2, 3 and 4 only
the provinces which were being claimed by c) 1, 2 and 4 only
the Muslim League as part of a separate d) 1, 3 and 4 only
nation i.e. Pakistan.
2. The Muslim league got the majority of Q.10) The CAPSTONE, recently seen in news, is
Muslim votes. a/an:
3. A Unionist-Muslim League-Akali coalition a) protein that can be used for genetic
under Khizr Hayat Khan assumed power in engineering.
Punjab. b) nano satellite launched by NASA.
Which of the statements given above is/are c) important geological feature of Jurassic
correct? period.
a) 1 only d) medicine that can be used to treat Ebola
b) 1 and 3 only virus.
c) 2 only
d) 2 and 3 only Q.11) Regarding Wood's Dispatch, which of the
following statements are true?
Q.8) Consider the following statements 1. Grants-in-Aid system was introduced.
regarding the various acts on press regulation: 2. Establishment of universities was
1. In Vernacular Press Act (1878), the recommended.
magistrate’s action was final, and no appeal 3. English as a medium of instruction at all
could levels of education was recommended.
be made in a court of law. Select the correct answer using the code given
2. In Indian Press Act (1910), the local below:
government was authorized to forfeit an a) 1 and 2 only
offending newspaper. b) 2 and 3 only
Which of the statements given above is/are c) 1 and 3 only
correct? d) 1, 2 and 3
a) 1 only
b) 2 only Q.12) Which of the following was/were the
c) Both 1 and 2 proposals of the Cabinet Mission plan?
d) Neither 1 nor 2 1. It proposed the partition of India into two
nations, India and Pakistan.
Q.9) With reference to C. Rajagopalachari (CR) 2. The Princely states would no longer be
Formula, consider the following statements: under the paramountcy of the British
1. It asked Muslim league to form a provisional Government.
government with the Indian National 3. It had proposed a two-tier executive and
Congress. legislature at provincial and union levels.
2. It proposed a plebiscite in the North-West Select the correct answer using the code given
and North-East India for creation of a below:
separate nation. a) 1 and 2 only
3. Mohammed Ali Jinnah opposed the plan as b) 1 and 3 only
he wanted the Congress to accept the Two c) 2 and 3 only
Nation Theory. d) 2 only
4. Mahatma Gandhi condemned the CR Plan as
it indirectly accepted the League’s demand
for separate nation.

Forum Learning Centre: Delhi - 2nd Floor, IAPL House, 19 Pusa Road, Karol Bagh, New Delhi - 110005 | Patna - 2nd floor, AG Palace, E Boring Canal
Road, Patna, Bihar 800001 | Hyderabad - 1st & 2nd Floor, SM Plaza, RTC X Rd, Indira Park Road, Jawahar Nagar, Hyderabad, Telangana 500020
9821711605 | https://academy.forumias.com | admissions@forumias.academy | helpdesk@forumias.academy
Page 4

SFG 2023 |LEVEL 1 |Test 22 |

Q.7) 1946 िे भारतीय प्रां तीय चुिार्ों िे सं र्दभट में , ऊपर नर्दए गए िथिों में से िौि से सही हैं ?
निम्ननिखित िथिों पर नर्चार िरें : a) िेर्ि 1, 2 और 3
1. िां ग्रेस निसी भी प्रां त में बहुमत हानसि िरिे में b) िेर्ि 2, 3 और 4
नर्फि रही, नजस पर मुखिम िीग द्वारा एि अिग c) िेर्ि 1, 2 और 4
राष्ट्र यािी पानिस्ताि िे नहस्से िे रूप में र्दार्ा निया d) िेर्ि 1, 3 और 4
जा रहा था।
2. मुखिम िीग िो अनधिां ि मुखिम र्ोर् नमिे। Q.10) हाि ही में समाचारों में र्दे िा गया CAPSTONE है :
3. खिज्र हयात िाि िे िेतृत्व में यू नियनिस्ट्-मुखिम a) प्रोर्ीि नजसिा उपयोग जेिेनर्ि इं जीनियररं ग िे निए
िीग-अिािी गठबं धि िे पंजाब में सत्ता सं भािी। निया जा सिता है ।
ऊपर नर्दए गए िथिों में से िौि सा/से सही है /हैं ? b) िासा द्वारा िॉन्च निया गया िैिो उपग्रह।
a) िेर्ि 1 c) जुरानसि िाि िी महत्वपूणट भू र्ैज्ञानिि नर्िेर्षता।
b) िेर्ि 1 और 3 d) र्ह र्दर्ा नजसिा उपयोग इबोिा र्ायरस िे इिाज िे
c) िेर्ि 2 निए निया जा सिता है ।
d) िेर्ि 2 और 3
Q.11) र्ु ि्स निस्पैच िे सं बंध में, निम्ननिखित में से िौि
Q.8) प्रेस नर्नियमि पर नर्नभन्न अनधनियमों िे सं बंध में से िथि सत्य हैं ?
निम्ननिखित िथिों पर नर्चार िरें : 1. अिुर्दाि सहायता प्रणािी िुरू िी गई।
1. र्िाट क्युिर प्रेस एक्ट (1878) में , मनजस्ट्रेर् िी िारट र्ाई 2. नर्श्वनर्द्याियों िी स्थापिा िी नसफाररि िी गई।
अंनतम थी, और न्यायािय में िोई अपीि िही ं िी जा 3. निक्षा िे सभी स्तरों पर निक्षा िे माध्यम िे रूप में
सिती थी। अंग्रेजी िी नसफाररि िी गई।
2. भारतीय प्रेस अनधनियम (1910) में, स्थािीय सरिार िीचे नर्दए गए िूर् िा प्रयोग िर सही उत्तर चुनिए:
िो नियमों िा उल्ंघि िरिे र्ािा समाचार पत्र िो
a) िेर्ि 1 और 2
जब्त िरिे िे निए अनधिृत निया गया।
b) िेर्ि 2 और 3
ऊपर नर्दए गए िथिों में से िौि सा/से सही है /हैं ?
c) िेर्ि 1 और 3
a) िेर्ि 1
d) 1, 2 और 3
b) िेर्ि 2
c) 1 और 2 र्दोिों
Q.12) निम्ननिखित में से िौि सा/से प्रस्तार् िैनबिेर्
d) ि तो 1 और ि ही 2
नमिि योजिा िे प्रस्तार् थे/थे ?
1. इसिे भारत िो र्दो राष्ट्रों, भारत और पानिस्ताि में
Q.9) सी राजगोपािाचारी (CR) फॉमूटिा िे सं र्दभट में, नर्भानजत िरिे िा प्रस्तार् रिा।
निम्ननिखित िथिों पर नर्चार िरें : 2. ररयासतें अब निनर्ि सरिार िी सर्ोच्चता िे अधीि
1. इसिे मुखिम िीग िो भारतीय राष्ट्रीय िां ग्रेस िे साथ िही ं होंगी।
एि अस्थायी सरिार बिािे िे निए िहा। 3. इसिे प्रां तीय और सं घ स्तरों पर र्दो स्तरीय िायट िारी
2. इसिे एि अिग राष्ट्र िे निमाट ण िे निए उत्तर-पनिम और नर्धानयिा िा प्रस्तार् नर्दया था।
और उत्तर-पूर्ट भारत में जिमत सं ग्रह िा प्रस्तार्
िीचे नर्दए गए िोि िा उपयोग िरिे सही उत्तर िा चयि
रिा।
िरें :
3. मोहम्मर्द अिी नजन्ना िे इस योजिा िा नर्रोध निया
a) िेर्ि 1 और 2
क्योंनि र्ह चाहते थे नि िां ग्रेस नद्व-राष्ट्र नसद्धां त िो
b) िेर्ि 1 और 3
स्वीिार िरे ।
c) िेर्ि 2 और 3
4. महात्मा गां धी िे CR योजिा िी निंर्दा िी क्योंनि इसिे
d) िेर्ि 2
अप्रत्यक्ष रूप से िीग िी अिग राष्ट्र िी मां ग िो
स्वीिार िर निया।

Forum Learning Centre: Delhi - 2nd Floor, IAPL House, 19 Pusa Road, Karol Bagh, New Delhi - 110005 | Patna - 2nd floor, AG Palace, E Boring Canal
Road, Patna, Bihar 800001 | Hyderabad - 1st & 2nd Floor, SM Plaza, RTC X Rd, Indira Park Road, Jawahar Nagar, Hyderabad, Telangana 500020
9821711605 | https://academy.forumias.com | admissions@forumias.academy | helpdesk@forumias.academy
Page 5

SFG 2023 |LEVEL 1 |Test 22 |

Q.13) Consider the following statements with Q.16) Consider the following statements
respect to the Mountbatten plan: regarding the Boundary commissions set up to
1. Under the plan, referendum was to be draw borders of India and Pakistan after
conducted in the North West Frontier partition:
Province and Sylhet district of Bengal. 1. Sir Cyril John Radcliffe was made the
2. Under the plan, a boundary commission was Chairman of two boundary commissions of
to be set up if partition was to be effected. Punjab and Bengal.
3. Under the plan, there was provision for two 2. The boundary commissions included two
Constituent Assemblies if partition was to nominees each of the Indian National
be affected. Congress and Muslim League respectively.
4. The Plan was opposed by both Congress and 3. The award of the Boundary Commissions
Muslim League. was announced before India declared its
Which of the statements given above are independence.
correct? Which of the statements given above is/are
a) 1 and 2 only correct?
b) 2, 3 and 4 only a) 1 and 2 only
c) 1, 2 and 3 only b) 2 only
d) 1, 2, 3 and 4 c) 1 and 3 only
d) 2 and 3 only
Q.14) Consider the following statements
regarding India Independence Act of 1947: Q.17) She was the first woman to contest for a
1. The Constituent Assembly of India which legislative seat in India, in the Madras provincial
also worked as the provisional parliament elections. She joined Indian National Congress
enacted the India Independence Act, 1947. in 1927. During the Salt March to Dandi, she
2. Under the act, both the Dominions of India convinced Gandhi to give equal opportunity to
and Pakistan, will have Governor Generals women to be in the forefront of the march.
appointed by Britain’s King/Queen. Later, she joined Seva Dal and trained women
3. The office of the Secretary of State for India activists.
will be continued till the adoption of the Which of the following personality is
Indian constitution. mentioned above?
Which of the statements given above is/are a) Kamaladevi Chattopadhyay
correct? b) Sarojini Naidu
a) 2 only c) Vijaya Lakshmi Pandit
b) 1 only d) Kamala Nehru
c) 1 and 3 only
d) 1 and 2 only Q.18) Consider the following pairs regarding the
Acts on development of Indian Press:
Q.15) Which of the following best describes the Act related to Provision
DeFi (Decentralized Financing) system, that has press
been recently seen in news? 1. Censorship of Every newspaper should
a) a system that provides a range of financial Press Act, 1799 print the names of the
services using digital assets. printers.
b) a technology used for the mining of crypto 2. Licensing Act, Replaced Metcalfe’s Act of
currencies. 1857 1835
c) a new loan disbursal scheme launched by 3. Registration Act, Copy of book to be
the RBI for achieving financial inclusion. 1867 submitted to the local
d) a new type of Non-Banking Financial government
Company set up to provide credit facilities How many pairs given above is/are correct?
in rural and tribal areas. a) Only one pair
b) Only two pairs
c) All three pairs
d) None of the pairs

Forum Learning Centre: Delhi - 2nd Floor, IAPL House, 19 Pusa Road, Karol Bagh, New Delhi - 110005 | Patna - 2nd floor, AG Palace, E Boring Canal
Road, Patna, Bihar 800001 | Hyderabad - 1st & 2nd Floor, SM Plaza, RTC X Rd, Indira Park Road, Jawahar Nagar, Hyderabad, Telangana 500020
9821711605 | https://academy.forumias.com | admissions@forumias.academy | helpdesk@forumias.academy
Page 6

SFG 2023 |LEVEL 1 |Test 22 |

Q.13) माउं र्बे र्ि योजिा िे सं बंध में निम्ननिखित िथिों Q.16) नर्भाजि िे बार्द भारत और पानिस्ताि िी
पर नर्चार िरें : सीमाओं िो निधाट ररत िरिे िे निए गनठत सीमा आयोगों
1. योजिा िे तहत उत्तर पनिम सीमां त प्रां त और बं गाि िे बारे में निम्ननिखित िथिों पर नर्चार िरें :
िे नसिहर् नजिे में जिमत सं ग्रह िराया जािा था। 1. सर नसररि जॉि रे िखिफ िो पं जाब और बं गाि िे
2. योजिा िे तहत, नर्भाजि प्रभानर्त होिे पर एि सीमा र्दो सीमा आयोगों िा अध्यक्ष बिाया गया।
आयोग िा गठि निया जािा था। 2. सीमा आयोगों में क्रमिः भारतीय राष्ट्रीय िां ग्रेस और
3. योजिा िे तहत, नर्भाजि प्रभानर्त होिे पर र्दो मुखिम िीग िे र्दो-र्दो िामां नित व्यखक्त िानमि थे।
सं नर्धाि सभाओं िा प्रार्धाि था। 3. भारत द्वारा अपिी स्वतं त्रता िी घोर्षणा िरिे से पहिे
4. इस योजिा िा िां ग्रेस और मुखिम िीग र्दोिों िे सीमा आयोगों िे पुरस्कार िी घोर्षणा िी गई थी।
नर्रोध निया था। ऊपर नर्दए गए िथिों में से िौि सा/से सही है /हैं ?
ऊपर नर्दए गए िथिों में से िौि सा सही है ? a) िेर्ि 1 और 2
a) िेर्ि 1 और 2 b) िेर्ि 2
b) िेर्ि 2, 3 और 4 c) िेर्ि 1 और 3
c) िेर्ि 1, 2 और 3 d) िेर्ि 2 और 3
d) 1, 2, 3 और 4
Q.17) र्ह मद्रास प्रां तीय चुिार्ों में भारत िी नर्धायी सीर्
Q.14) 1947 िे भारत स्वतं त्रता अनधनियम िे बारे में िे निए चु िार् िड़िे र्ािी पहिी मनहिा थी।ं र्ह 1927
निम्ननिखित िथिों पर नर्चार िरें : में भारतीय राष्ट्रीय िां ग्रेस में िानमि हुईं। र्दां िी में िमि
1. भारत िी सं नर्धाि सभा, नजसिे अस्थायी सं सर्द िे माचट िे र्दौराि, उन्ोंिे माचट में गां धी िो मनहिाओं िो
रूप में भी िाम निया, िे भारत स्वतं त्रता अनधनियम, समाि अर्सर र्दे िे िे निए राजी निया। बार्द में, र्ह से र्ा
1947 िो अनधनियनमत निया। र्दि में िानमि हो गईं और मनहिा िायट िताट ओं िो
2. अनधनियम िे तहत, भारत और पानिस्ताि र्दोिों प्रनिनक्षत निया।
िोनमनियि में, निर्े ि िे सम्रार्/सम्राज्ञी द्वारा नियु क्त निम्ननिखित में से निस व्यखक्तत्व िा उल्ेि ऊपर निया
गर्िटर जिरि होंगे। गया है ?
3. भारत िे राज्य सनचर् िा िायाट िय भारतीय सं नर्धाि a) िमिार्दे र्ी चट्टोपाध्याय
िो अपिािे ति जारी रहे गा। b) सरोनजिी िायिू
ऊपर नर्दए गए िथिों में से िौि सा/से सही है /हैं ? c) नर्जया िक्ष्मी पंनित
a) िेर्ि 2 d) िमिा िेहरू
b) िेर्ि 1
c) िेर्ि 1 और 3 Q.18) भारतीय प्रेस िे नर्िास सं बंधी अनधनियमों िे
d) िेर्ि 1 और 2 सं बंध में निम्ननिखित यु ग्ों पर नर्चार िीनजए:
प्रेस से संबंधित प्राविाि
Q.15) निम्ननिखित में से िौि DeFi (नर्िेंद्रीिृत अधिधियम
नर्त्तपोर्षण) प्रणािी िा सबसे अच्छा र्णट ि िरता है , नजसे 1. प्रेस अनधनियम, 1799 प्रत्येि समाचार पत्र िो
िी सें सरनिप मुद्रिों िा िाम छापिा
हाि ही में समाचारों में र्दे िा और सु िा गया है ?
चानहए।
a) एि प्रणािी जो निनजर्ि सं पनत्त िा उपयोग िरिे
2. िाइसें नसं ग अनधनियम, 1835 िे मेर्िाफ
नर्त्तीय से र्ाओं िी एि श्ृंििा प्रर्दाि िरती है । 1857 अनधनियम िो बर्दिा
b) नक्रप्टो मुद्राओं िे मीनिंग िे निए उपयोग िी जािे गया
र्ािी तििीि। 3. पंजीिरण अनधनियम, स्थािीय सरिार िो
c) नर्त्तीय समार्े िि प्राप्त िरिे िे निए RBI द्वारा िुरू 1867 प्रस्तु त िी जािे र्ािी
िी गई एि िई ऋण नर्तरण योजिा। पुस्ति िी प्रनत
ऊपर नर्दए गए नितिे यु ग् सही है /हैं ?
d) ग्रामीण और आनर्दर्ासी क्षे त्रों में ऋण सु नर्धा प्रर्दाि
a) िेर्ि एि जोड़ी
िरिे िे निए एि िए प्रिार िी गै र -बैं निंग नर्त्तीय
b) िेर्ि र्दो जोड़े
िंपिी स्थानपत िी गई।
c) तीिों जोड़े
d) िोई भी जोड़ा िही ं

Forum Learning Centre: Delhi - 2nd Floor, IAPL House, 19 Pusa Road, Karol Bagh, New Delhi - 110005 | Patna - 2nd floor, AG Palace, E Boring Canal
Road, Patna, Bihar 800001 | Hyderabad - 1st & 2nd Floor, SM Plaza, RTC X Rd, Indira Park Road, Jawahar Nagar, Hyderabad, Telangana 500020
9821711605 | https://academy.forumias.com | admissions@forumias.academy | helpdesk@forumias.academy
Page 7

SFG 2023 |LEVEL 1 |Test 22 |

Q.19) With reference to Subhash Chandra Bose c) The minute was based on the orientalist
and Indian National Army (INA), consider the view on education.
following statements: d) The minute aimed to help in achieving the
1. Subhash Chandra Bose opposed the Quit Civilizing mission of the British
India Movement as yet another ineffective Government.
nonviolent struggle.
2. The defeat of British by INA during Imphal Q.23) Which of the following actions/policies
campaign encouraged its members to by the British were responsible for the creation
liberate India from British. of Pakistan as a separate nation?
Which of the statements given above is/are 1. The Cripps mission’s proposal provided
correct? legitimacy to the demand for the creation of
a) 1 only a separate nation.
b) 2 only 2. Recognition of Muslim league as a sole
c) Both 1 and 2 representative of Muslims in India by
d) Neither 1 nor 2 Wavell’s plan.
3. The British invited the Congress to form
Q.20) Which one of the statements given below interim government without taking into
most appropriately describes the term ‘Gene consideration the Muslim League’s
Therapy’? demands.
a) It is a type of medical test that is used in 4. Acceptance of Pakistan as a separate nation
identification of changes in genes, by the Cabinet mission plan.
chromosomes, or proteins. Select the correct answer using the code given
b) It is a technique of enhancing capabilities of below:
an organism beyond the normal levels a) 1 and 2 only
through genetic modifications. b) 1 and 3 only
c) It is the ability of genetic improvement c) 2 and 3 only
through the correction of altered genes or d) 1, 3 and 4 only
site-specific modifications.
d) It is a technique that mainly aims at the Q.24) Consider the following statements
prevention of communicable diseases regarding the development of Education under
through DNA analysis. the British rule in India:
1. Under the 1913 Resolution on Education
Q.21) The Vital-Vidhvansak, the first monthly policy, the British government agreed to
journal to have the untouchable people as its provide free primary education to all.
target audience was published by 2. In 1902, Raleigh Commission was set up to
a) Gopal Baba Walangkar go into conditions and prospects of
b) Jyotiba Phule universities in India.
c) Mohandas Karamchand Gandhi 3. The Sergeant plan of 1944 provided for
d) Bhimrao Ramji Ambedkar technical, commercial and arts education.
Which of the statements given above are
Q.22) T B Macaulay's Minute on Indian correct?
Education of 1835 is regarded by many as the a) 1 and 2 only
most significant document in the history of b) 1 and 3 only
Indian education. In this context, which of the c) 2 and 3 only
following statements is incorrect about this d) 1, 2 and 3
Macaulay’s Minute?
a) The minute rejected indigenous literatures
and languages and promoted English.
b) The minute encapsulates the concept of the
White man's burden.

Forum Learning Centre: Delhi - 2nd Floor, IAPL House, 19 Pusa Road, Karol Bagh, New Delhi - 110005 | Patna - 2nd floor, AG Palace, E Boring Canal
Road, Patna, Bihar 800001 | Hyderabad - 1st & 2nd Floor, SM Plaza, RTC X Rd, Indira Park Road, Jawahar Nagar, Hyderabad, Telangana 500020
9821711605 | https://academy.forumias.com | admissions@forumias.academy | helpdesk@forumias.academy
Page 8

SFG 2023 |LEVEL 1 |Test 22 |

Q.19) सु भार्ष चंद्र बोस और आज़ार्द नहन्द फौज c) नर्र्रण पत्र निक्षा पर प्राच्यर्ार्दी दृनष्ट्िोण पर
(आईएिए) िे सं र्दभट में निम्ननिखित िथिों पर नर्चार िरें : आधाररत था।
1. सु भार्ष चंद्र बोस िे भारत छोड़ो आं र्दोिि िा एि और d) इस नर्र्रण पत्र िा उद्दे श्य निनर्ि सरिार िे सभ्यता
अप्रभार्ी अनहं सि सं घर्षट मािते हुए नर्रोध निया। नमिि िो प्राप्त िरिे में मर्दर्द िरिा था।
2. इं फाि अनभयाि िे र्दौराि आईएिए द्वारा अंग्रेजों िी
हार िे इसिे सर्दस्ों िो भारत िो अंग्रे जों से स्वतं त्र Q.23) पानिस्ताि िो एि अिग राष्ट्र िे रूप में बिािे
िरिे िे निए प्रोत्सानहत निया। िे निए अंग्रेजों द्वारा निम्ननिखित में से िौि सी िा
ऊपर नर्दए गए िथिों में से िौि सा/से सही है /हैं ? रट र्ाइयां /िीनतयां नजम्मेर्दार थी?ं
a) िेर्ि 1 1. नक्रप्स नमिि िे प्रस्तार् िे एि अिग राष्ट्र िे निमाट ण
b) िेर्ि 2 िी मां ग िो र्ै धता प्रर्दाि िी।
c) 1 और 2 र्दोिों 2. र्े र्ेि िी योजिा द्वारा मुखिम िीग िो भारत में
d) ि तो 1 और ि ही 2 मुसिमािों िे एिमात्र प्रनतनिनध िे रूप में मान्यता
र्दी गयी।
Q.20) िीचे नर्दए गए िथिों में से िौि सा 'जीि थेरेपी' 3. अंग्रेजों िे मुखिम िीग िी मां गों पर नर्चार निए नबिा
िब्द िा सबसे उपयु क्त र्णट ि िरता है ? िां ग्रेस िो अंतररम सरिार बिािे िे निए आमंनत्रत
a) यह एि प्रिार िा नचनित्सीय परीक्षण है नजसिा निया।
उपयोग जीि, क्रोमोसोम या प्रोर्ीि में पररर्तट ि िी 4. िैनबिेर् नमिि योजिा द्वारा पानिस्ताि िो एि
पहचाि िे निए निया जाता है । अिग राष्ट्र िे रूप में स्वीिार िरिा।
b) यह आिुर्ंनिि सं िोधिों िे माध्यम से सामान्य स्तरों िीचे नर्दए गए िूर् िा प्रयोग िर सही उत्तर चुनिए:
से परे एि जीर् िी क्षमताओं िो बढ़ािे िी एि a) िेर्ि 1 और 2
तििीि है । b) िेर्ि 1 और 3
c) यह पररर्नतट त जीिों या साइर्-नर्निष्ट् सं िोधिों िे c) िेर्ि 2 और 3
सु धार िे माध्यम से आिुर्ंनिि सु धार िी क्षमता है । d) िेर्ि 1, 3 और 4
d) यह एि ऐसी तििीि है नजसिा मुख्य उद्दे श्य
िीएिए नर्श्ले र्षण िे माध्यम से सं चारी रोगों िी Q.24) भारत में निनर्ि िासि िे तहत निक्षा िे नर्िास
रोिथाम िरिा है । िे सं बंध में निम्ननिखित िथिों पर नर्चार िरें :
1. निक्षा िीनत पर 1913 िे सं िल्प िे तहत, निनर्ि
Q.21) अस्पृश्य िोगों िो अपिे िनक्षत पाठि र्गट िे रूप सरिार सभी िो मुफ्त प्राथनमि निक्षा प्रर्दाि िरिे
में रििे र्ािी पहिी मानसि पनत्रिा र्द र्ाइर्ि-नर्धं सि िे निए सहमत हुई।
निसिे द्वारा प्रिानित िी गई थी? 2. 1902 में, भारत में नर्श्वनर्द्याियों िी खस्थनतयों और
a) गोपाि बाबा र्ािंगिर सं भार्िाओं में जािे िे निए रै िे आयोग िी स्थापिा
b) ज्योनतबा फुिे िी गई थी।
c) मोहिर्दास िरमचंर्द गां धी 3. 1944 िी साजेंर् योजिा तििीिी, र्ानणखज्यि और
d) भीमरार् रामजी अम्बे ििर ििा निक्षा िे निए प्रर्दाि िी गई।
ऊपर नर्दए गए िथिों में से िौि से सही हैं ?
Q.22) 1835 िे भारतीय निक्षा पर र्ी बी मैिािे िा a) िेर्ि 1 और 2
नर्र्रण पत्र िो भारतीय निक्षा िे इनतहास में सबसे b) िेर्ि 1 और 3
महत्वपूणट र्दस्तार्े ज मािा जाता है । इस सं र्दभट में, मैिािे c) िेर्ि 2 और 3
िे इस नर्र्रण पत्र िे बारे में निम्ननिखित में से िौि सा d) 1, 2 और 3
िथि गित है ?
a) नर्र्रण पत्र िे स्वर्दे िी सानहत्य और भार्षाओं िो
िाररज िर नर्दया और अंग्रेजी िो बढ़ार्ा नर्दया।
b) नर्र्रण पत्र श्वे त व्यखक्त िे बोझ िी अर्धारणा िो
समानहत िरता है ।

Forum Learning Centre: Delhi - 2nd Floor, IAPL House, 19 Pusa Road, Karol Bagh, New Delhi - 110005 | Patna - 2nd floor, AG Palace, E Boring Canal
Road, Patna, Bihar 800001 | Hyderabad - 1st & 2nd Floor, SM Plaza, RTC X Rd, Indira Park Road, Jawahar Nagar, Hyderabad, Telangana 500020
9821711605 | https://academy.forumias.com | admissions@forumias.academy | helpdesk@forumias.academy
Page 9

SFG 2023 |LEVEL 1 |Test 22 |

Q.25) With reference to the Panchayats Which of the statements given above is/are
Extension to The Scheduled Areas (PESA) Act of correct?
1996, consider the following statements: a) 1 and 2 only
1. The act aims to extend the Constitutional b) 2 and 3 only
provisions of Panchayats to Fifth and Sixth c) 1 only
scheduled areas. d) 1, 2 and 3
2. Chhattisgarh was the first state in India to
implement the provisions of PESA act. Q.29) What was the immediate reason leading
3. Rules regarding the implementation of the to Muhammad Ali Jinnah’s declaration of 16
act can be framed by the Centre alone and August, 1946 as Direct Action Day?
not by the states. a) Rejection of demand to form full-fledged
Which of the above statements is/are Pakistan under Cabinet Mission plan.
incorrect? b) Nehru’s statement that the constituent
a) 1 and 3 only assembly would not have the compulsory
b) 1 only grouping of provincial assemblies.
c) 2 and 3 only c) Congress formed the interim government
d) 1, 2 and 3 without accepting the demands put forward
by Muslim league.
Q.26) Which of the following statements is d) Overthrowing the coalition government in
correct with reference to the provisions of the Punjab.
Wavell’s Breakdown plan?
a) Except governor- general and commander- Q.30) Consider the following pair of places and
in-chief, all other members of the executive their location:
council to be Indians. Place Location
b) Transfer of power to separate provinces and 1. Al-Aqsa Mosque Turkey
an option given to remain independent. 2. Katchal Island Sri Lanka
c) Punjab and Bengal legislative assemblies 3. Lake Garda Italy
would meet in two groups of Hindus and Which of the above pairs is/are correctly
Muslims to vote for the partition of their matched?
provinces.
a) 2 only
d) Withdrawal of British army and British
b) 3 only
officials to the Muslim provinces of north
c) 1 and 2 only
west and north east of India.
d) 1, 2 and 3

Q.27) The National Defence Council was set up


Q.31) With reference to Indian History, the
as a result of which of the following events?
Members of the Constituent Assembly from the
a) Indian National Session of Haripura
Provinces were:
b) Cripps Mission
a) directly elected by the people of those
c) August Offer Provinces
d) Indian National Session of Faizpur b) nominated by the Indian National Congress
and the Muslim League
Q.28) With reference to interim government of c) elected by the Provincial Legislative
1946, consider the following statements: Assemblies
1. Muslim league members joined the interim d) selected by the Government for their
government with the condition of not giving expertise in constitutional matters
up with their ‘direct action’ program.
2. It functioned according to the provision of
Government of India Act of 1919.
3. Jawaharlal Nehru was the President of the
Viceroys Executive Council.

Forum Learning Centre: Delhi - 2nd Floor, IAPL House, 19 Pusa Road, Karol Bagh, New Delhi - 110005 | Patna - 2nd floor, AG Palace, E Boring Canal
Road, Patna, Bihar 800001 | Hyderabad - 1st & 2nd Floor, SM Plaza, RTC X Rd, Indira Park Road, Jawahar Nagar, Hyderabad, Telangana 500020
9821711605 | https://academy.forumias.com | admissions@forumias.academy | helpdesk@forumias.academy
Page 10

SFG 2023 |LEVEL 1 |Test 22 |

Q.25) 1996 िे अिुसूनचत क्षे त्रों (PESA) अनधनियम िे 3. जर्ाहरिाि िे हरू र्ायसराय िी िायट िारी पररर्षर्द
पंचायत नर्स्तार िे सं र्दभट में, निम्ननिखित िथिों पर िे अध्यक्ष थे।
नर्चार िरें : ऊपर नर्दए गए िथिों में से िौि सा/से सही है /हैं ?
1. अनधनियम िा उद्दे श्य पं चायतों िे सं र्ैधानिि a) िेर्ि 1 और 2
प्रार्धािों िो पां चर्ी ं और छठी अिुसूनचत क्षे त्रों ति b) िेर्ि 2 और 3
नर्स्ताररत िरिा है ।
c) िेर्ि 1
2. पेसा अनधनियम िे प्रार्धािों िो िागू िरिे र्ािा
d) 1, 2 और 3
छत्तीसगढ़ भारत िा पहिा राज्य था।
3. अनधनियम िे िायाट न्वयि िे नियम अिेिे िेंद्र द्वारा
Q.29) मुहम्मर्द अिी नजन्ना द्वारा 16 अगस्त, 1946 िो
बिाए जा सिते हैं , ि नि राज्यों द्वारा।
प्रत्यक्ष िारट र्ाई नर्दर्स घोनर्षत िरिे िा तात्कानिि
उपरोक्त िथिों में से िौि सा/से गित है /हैं ?
िारण क्या था?
a) िेर्ि 1 और 3
a) िैनबिेर् नमिि योजिा िे तहत पू णट पानिस्ताि बिािे
b) िेर्ि 1
िी मां ग िो िाररज िरिा।
c) िेर्ि 2 और 3
b) िेहरू िा यह िथि नि सं नर्धाि सभा में प्रां तीय
d) 1, 2 और 3
नर्धािसभाओं िा अनिर्ायट समूहीिरण िही ं होिा।
c) मुखिम िीग द्वारा रिी गई मां गों िो स्वीिार निए
Q.26) र्े र्ेल्स िे ििाउि योजिा िे प्रार्धािों िे सं र्दभट में
नबिा िां ग्रेस िा अंतररम सरिार बिािा।
निम्ननिखित में से िौि सा िथि सही है ?
d) पंजाब में गठबं धि सरिार िो उिाड़ फेंििा।
a) गर्िटर-जिरि और िमां िर-इि-चीफ िो छोड़िर,
िायट िारी पररर्षर्द िे अन्य सभी सर्दस् भारतीय होंगे।
Q.30) निम्ननिखित स्थािों और उििे स्थाि पर नर्चार
b) अिग प्रां तों िो सत्ता िा हस्तां तरण और स्वतं त्र रहिे
िरें :
िा नर्िल्प नर्दया गया।
स्थाि अवस्स्थधत
c) पंजाब और बं गाि नर्धािसभाएं अपिे प्रां तों िे
नर्भाजि िे निए मतर्दाि िरिे िे निए नहं र्दुओं और 1.अि-अक्सा मखिर्द तु िी
मुसिमािों िे र्दो समूहों में नमिेंगी। 2. िच्छि द्वीप श्ीिंिा
d) भारत िे उत्तर पनिम और उत्तर पूर्ट िे मुखिम प्रां तों 3. गािाट झीि इर्िी
में निनर्ि से िा और निनर्ि अनधिाररयों िी र्ापसी।
उपरोक्त यु ग्ों में से िौि-सा/से सही सु मेनित है /हैं ?
a) िेर्ि 2
Q.27) निम्ननिखित में से निस घर्िा िे पररणामस्वरूप
b) िेर्ि 3
राष्ट्रीय रक्षा पररर्षर्द िी स्थापिा िी गई थी?
c) िेर्ि 1 और 2
a) हररपुरा िा भारतीय राष्ट्रीय अनधर्े िि
d) 1, 2 और 3
b) नक्रप्स नमिि
c) अगस्त प्रस्तार्
Q.31) भारतीय इनतहास िे संर्दभट में प्रां तों से सं नर्धाि
d) फैजपुर िा भारतीय राष्ट्रीय अनधर्े िि
सभा िे सर्दस् थे:
a) सीधे उि प्रां तों िे िोगों द्वारा चुिे गए
Q.28) 1946 िी अंतररम सरिार िे संर्दभट में
b) भारतीय राष्ट्रीय िां ग्रेस और मुखिम िीग द्वारा िानमत
निम्ननिखित िथिों पर नर्चार िीनजये :
c) प्रां तीय नर्धाि सभाओं द्वारा निर्ाट नचत
1. मुखिम िीग िे सर्दस् अपिे 'िायरे क्ट एक्शि'
िायट क्रम िो ि छोड़िे िी ितट िे साथ अंतररम d) सं र्ैधानिि मामिों में उििी नर्िेर्षज्ञता िे निए

सरिार में िानमि हुए। सरिार द्वारा चयनित

2. इसिे 1919 िे भारत सरिार अनधनियम िे प्रार्धाि


िे अिुसार िायट निया।

Forum Learning Centre: Delhi - 2nd Floor, IAPL House, 19 Pusa Road, Karol Bagh, New Delhi - 110005 | Patna - 2nd floor, AG Palace, E Boring Canal
Road, Patna, Bihar 800001 | Hyderabad - 1st & 2nd Floor, SM Plaza, RTC X Rd, Indira Park Road, Jawahar Nagar, Hyderabad, Telangana 500020
9821711605 | https://academy.forumias.com | admissions@forumias.academy | helpdesk@forumias.academy
Page 11

SFG 2023 |LEVEL 1 |Test 22 |

Q.32) With reference to Congress’s stand 2. As per RBI, generally the payment data shall
towards Cabinet Mission proposals, consider be stored in systems located in India.
the following statements: 3. Ministry of Home Affairs launched ‘Cyber
1. Congress interpreted Cabinet Mission Surakshith Bharat’ initiative to raise
proposals as mandatory grouping of awareness about the cyber-crimes.
provincial assemblies into three sections. Which of the statements given above is/are
2. Congress all together rejected the proposals correct?
under Cabinet mission. a) 1 and 2 only
3. Congress objected to the proposal that b) 3 only
provinces need to wait till the first general c) 1, 2 and 3
elections to come out of a group. d) 1 only
Which of the statements given above is/are
correct? Q.36) With reference to the Constituent
a) 1 and 3 only Assembly of India, consider the following
b) 2 and 3 only statements:
c) 3 only 1. The Indian National Congress officially
d) 1 and 2 only demanded a Constituent Assembly for the
first time in 1935.
Q.33) With reference to the interim 2. The demand for the constituent assembly
Government of 1946, consider the following was accepted, in principle, by the British in
statements: August Offer.
1. It was formed from the Constituent 3. It was constituted under the scheme
Assembly which was elected in August 1946. formulated by Cripps Mission Plan.
2. Dr Babasaheb Ambedkar headed the Which of the statements given above is/are
portfolio of Law in the Interim government. correct?
3. It appointed a committee to advise the a) 1 and 2 only
government on nationalizing the armed b) 2 and 3 only
forces. c) 1 only
Which of the above statements is/are correct? d) 1, 2, and 3
a) 1 and 3 only
b) 2 and 3 only Q.37) Which of the following statements are
c) 2 only correct with reference to the Objective
d) 1 and 2 only Resolution introduced by Jawaharlal Nehru?
1. It laid down the general philosophy behind
Q.34) The primary objective of the Pirpur the Indian Constitution.
Committee was to: 2. It promises to provide adequate safeguards
a) prepare report on the atrocities supposedly for the minorities and backward sections of
committed by the Congress ministries society.
formed after 1937 elections. 3. India to have a centralized form of
b) resolve issues pertaining to minorities that government with a strong Centre.
has affected Indian political discourse. 4. It seeks to maintain the sovereign rights on
c) examine the governance reforms proposed land, sea, and air according to the law of
by the Government of India Act, 1919. civilized nations.
d) enquire into conditions of universities of Select the correct answer using the code given
India. below:
a) 1 and 2 only
Q.35) Consider the following statements with b) 2 and 3 only
reference to measures for Data protection in c) 2,3 and 4 only
India: d) 1, 2, and 4 only
1. CERT-In is the national nodal agency for
responding to cybersecurity threats.

Forum Learning Centre: Delhi - 2nd Floor, IAPL House, 19 Pusa Road, Karol Bagh, New Delhi - 110005 | Patna - 2nd floor, AG Palace, E Boring Canal
Road, Patna, Bihar 800001 | Hyderabad - 1st & 2nd Floor, SM Plaza, RTC X Rd, Indira Park Road, Jawahar Nagar, Hyderabad, Telangana 500020
9821711605 | https://academy.forumias.com | admissions@forumias.academy | helpdesk@forumias.academy
Page 12

SFG 2023 |LEVEL 1 |Test 22 |

Q.32) िैनबिेर् नमिि प्रस्तार्ों िे प्रनत िां ग्रेस िे रुि िे 2. भारतीय ररजर्ट बैं ि िे अिुसार, आम तौर पर भु गताि
सं र्दभट में, निम्ननिखित िथिों पर नर्चार िरें : िे र्ा भारत में खस्थत नसस्ट्म में संग्रहीत निया जाएगा।
1. िां ग्रेस िे िैनबिेर् नमिि िे प्रस्तार्ों िी व्याख्या 3. गृ ह मंत्रािय िे साइबर अपराधों िे बारे में जागरूिता
प्रां तीय नर्धािसभाओं िो तीि र्गों में अनिर्ायट रूप बढ़ािे िे निए 'साइबर सु रनक्षत भारत' पहि िुरू
से समूहीिृत िरिे िे रूप में िी। िी।
2. िां ग्रेस िे नमििर िैनबिेर् नमिि िे प्रस्तार्ों िो ऊपर नर्दए गए िथिों में से िौि सा/से सही है /हैं ?
िाररज िर नर्दया। a) िेर्ि 1 और 2
3. िां ग्रेस िे इस प्रस्तार् पर आपनत्त जताई नि प्रां तों िो b) िेर्ि 3
एि समूह से बाहर आिे िे निए पहिे आम चु िार् c) 1, 2 और 3
ति इं तजार िरिा होगा। d) िेर्ि 1
ऊपर नर्दए गए िथिों में से िौि सा/से सही है /हैं ?
a) िेर्ि 1 और 3 Q.36) भारत िी सं नर्धाि सभा िे सं र्दभट में निम्ननिखित
b) िेर्ि 2 और 3 िथिों पर नर्चार िीनजये :
c) िेर्ि 3 1. भारतीय राष्ट्रीय िां ग्रेस िे आनधिाररि तौर पर 1935
d) िेर्ि 1 और 2 में पहिी बार सं नर्धाि सभा िी मां ग िी।
2. अगस्त प्रस्तार् में निनर्ि द्वारा सं नर्धाि सभा िी मां ग
Q.33) 1946 िी अंतररम सरिार िे सं र्दभट में , िो सै द्धां नति रूप से स्वीिार िर निया गया था।
निम्ननिखित िथिों पर नर्चार िरें : 3. इसिा गठि नक्रप्स नमिि योजिा द्वारा तै यार योजिा
1. इसिा गठि अगस्त 1946 में चुिी गई सं नर्धाि सभा िे तहत निया गया था।
से हुआ था। ऊपर नर्दए गए िथिों में से िौि सा/से सही है /हैं ?
2. िॉ बाबासाहे ब अंबेििर िे अंतररम सरिार में िािूि a) िेर्ि 1 और 2
नर्भाग िा िेतृत्व निया। b) िेर्ि 2 और 3
3. इसिे सिस्त्र बिों िे राष्ट्रीयिरण पर सरिार िो c) िेर्ि 1
सिाह र्दे िे िे निए एि सनमनत नियु क्त िी। d) 1, 2, और 3
उपरोक्त िथिों में से िौि-सा/से सही है /हैं ?
a) िेर्ि 1 और 3 Q.37) निम्ननिखित में से िौि सा िथि जर्ाहरिाि
b) िेर्ि 2 और 3 िेहरू द्वारा पेि निए गए उद्दे श्य सं िल्प िे सं र्दभट में सही
c) िेर्ि 2 है ?
d) िेर्ि 1 और 2 1. इसिे भारतीय सं नर्धाि िे पीछे सामान्य र्दिटि
निधाट ररत निया।
Q.34) पीरपुर सनमनत िा प्राथनमि उद्दे श्य था: 2. यह अल्पसं ख्यिों और समाज िे नपछड़े र्गों िे निए
a) 1937 िे चुिार्ों िे बार्द गनठत िां ग्रेसी मंनत्रमंििों पयाट प्त सु रक्षा प्रर्दाि िरिे िा र्ार्दा िरता है ।
द्वारा िनथत तौर पर निए गए अत्याचारों पर ररपोर्ट 3. भारत में एि मजबू त िेंद्र िे साथ सरिार िा एि
तै यार िरिा। िेंद्रीिृत रूप होगा।
b) अल्पसं ख्यिों से सं बंनधत मुद्दों िो हि िरिा नजसिे 4. यह सभ्य राष्ट्रों िे िािू ि िे अिु सार भू नम, समुद्र और
भारतीय राजिीनति सं र्ार्द िो प्रभानर्त निया है । र्ायु पर सं प्रभु अनधिारों िो बिाए रििा चाहता है ।
c) भारत सरिार अनधनियम, 1919 द्वारा प्रस्तानर्त िीचे नर्दए गए िूर् िा प्रयोग िर सही उत्तर चुनिए:
िासि सु धारों िी जां च िरिा। a) िेर्ि 1 और 2
d) भारत िे नर्श्वनर्द्याियों िी खस्थनतयों िे बारे में b) िेर्ि 2 और 3
पूछताछ िरिा। c) िेर्ि 2, 3 और 4
d) िेर्ि 1, 2 और 4
Q.35) भारत में िे र्ा सु रक्षा िे उपायों िे सं र्दभट में
निम्ननिखित िथिों पर नर्चार िरें :
1. CERT-In साइबर सु रक्षा ितरों िा प्रनतनक्रया र्दे िे िे
निए राष्ट्रीय िोिि एजेंसी है ।

Forum Learning Centre: Delhi - 2nd Floor, IAPL House, 19 Pusa Road, Karol Bagh, New Delhi - 110005 | Patna - 2nd floor, AG Palace, E Boring Canal
Road, Patna, Bihar 800001 | Hyderabad - 1st & 2nd Floor, SM Plaza, RTC X Rd, Indira Park Road, Jawahar Nagar, Hyderabad, Telangana 500020
9821711605 | https://academy.forumias.com | admissions@forumias.academy | helpdesk@forumias.academy
Page 13

SFG 2023 |LEVEL 1 |Test 22 |

Q.38) The establishment of British territorial Q.41) With reference to the Cabinet Mission,
control over India brought changes in different which of the following statements is/are
spheres of life. Education was one of such areas correct?
where lot of changes came with the transfer of 1. It recommended a federal government.
power to the British. The British Government in 2. It enlarged the powers of the Indian courts.
India established many commissions in the field 3. It provided for more Indians in the ICS.
of education. Select the correct answer using the code given
Which of the Commissions/committee below.
mentioned below are related to the a) 1 only
development of Education? b) 2 and 3
1. Hunter Commission (1882-83) c) 1 and 3
2. Aitchison Committee (1886) d) None
3. Saddler Commission (1917-19)
4. Hartog Committee (1929) Q.42) With reference to the Unionist party,
5. Linlithgow Commission (1926) consider the following statements:
Select the correct answer using the code given 1. Sir Syed Ahmed Khan was one of the
below: prominent members of this party.
a) 1, 2, 3 and 4 only 2. The party mainly defended the rights of
b) 1, 3 and 4 only landless labourers in Punjab against the
c) 1, 3 and 5 only landlords.
d) 1, 2, 3, 4 and 5 3. The party supported Muslim League’s
demand for partition of India.
Q.39) With reference to the Muslim League’s Which of the statements given above is/are
Lahore Resolution of 1940, consider the correct?
following statements: a) 1 and 3 only
1. One of the demands was the creation of b) 3 only
‘Pakistan’ as a separate nation for Muslims. c) 2 only
2. The resolution was drafted by Sikandar d) None of the above
Hayat Khan.
Which of the statements given above is/are Q.43) With respect to the history of demand for
correct? creation of Pakistan as a separate nation,
a) 1 only consider the following statements
b) 2 only 1. Choudhry Rahmat Ali presented a pamphlet
c) Both 1 and 2 to the British during a round table
d) Neither 1 nor 2
conference, demanding a separate nation of
Pakistan.
Q.40) Consider the following statements
2. The demand of Pakistan as a separate nation
regarding ‘Ayushman Bharat Digital Mission
have been traced back to the Urdu poet
(ABDM)’:
Mohammad Iqbal.
1. The participation in the Mission is
3. The Launch of direct-action day by
mandatory for the citizens availing subsidies
Muhamad Ali Jinnah accentuated the
granted by Central government.
demand for Pakistan as a separate nation.
2. Under the mission, patients will have
Which of the statements given above are
opportunity to access their medical records
on their mobile phone. correct?
3. The medical records of the patients under a) 1 and 2 only
ABDM are stored up to a maximum period of b) 2 and 3 only
one year only. c) 1 and 3 only
Which of the statements given above is/are d) 1, 2 and 3
correct?
a) 1 only
b) 2 only
c) 2 and 3 only
d) 1, 2 and 3

Forum Learning Centre: Delhi - 2nd Floor, IAPL House, 19 Pusa Road, Karol Bagh, New Delhi - 110005 | Patna - 2nd floor, AG Palace, E Boring Canal
Road, Patna, Bihar 800001 | Hyderabad - 1st & 2nd Floor, SM Plaza, RTC X Rd, Indira Park Road, Jawahar Nagar, Hyderabad, Telangana 500020
9821711605 | https://academy.forumias.com | admissions@forumias.academy | helpdesk@forumias.academy
Page 14

SFG 2023 |LEVEL 1 |Test 22 |

Q.38) भारत पर निनर्ि क्षे त्रीय नियं त्रण िी स्थापिा िे Q.41) िैनबिेर् नमिि िे सं र्दभट में , निम्ननिखित में से
जीर्ि िे नर्नभन्न क्षे त्रों में पररर्तटि िाए। निक्षा उि क्षे त्रों िौि सा/से िथि सही है /हैं ?
में से एि था जहां अंग्रेजों िो सत्ता हस्तां तरण िे साथ 1. इसिे सं घीय सरिार िी नसफाररि िी।
बहुत सारे बर्दिार् आए। भारत में निनर्ि सरिार िे निक्षा 2. इसिे भारतीय न्यायाियों िी िखक्तयों िा नर्स्तार
िे क्षे त्र में िई आयोगों िी स्थापिा िी। िीचे नर्दए गए निया।
आयोगों/सनमनतयों में से िौि-सा निक्षा िे नर्िास से 3. इसिे आईसीएस में अनधि भारतीयों िे निए प्रार्धाि
सं बंनधत है ? निया।
1. हं र्र िमीिि (1882-83) िीचे नर्दए गए िूर् िा प्रयोग िर सही उत्तर चुनिए।
2. एचीसि सनमनत (1886) a) िेर्ि 1
3. सै ििर िमीिि (1917-19) b) 2 और 3
4. हार्ोग िमेर्ी (1929) c) 1 और 3
5. नििनिथगो आयोग (1926) d) िोई िही ं
िीचे नर्दए गए िूर् िा प्रयोग िर सही उत्तर चुनिए:
a) िेर्ि 1, 2, 3 और 4 Q.42) यूनियनिस्ट् पार्ी िे सं र्दभट में निम्ननिखित िथिों
b) िेर्ि 1,3 और 4 पर नर्चार िीनजये :
c) िेर्ि 1, 3 और 5 1. सर सै यर्द अहमर्द िाि इस पार्ी िे प्रमुि सर्दस्ों में
d) 1, 2, 3, 4 और 5 से एि थे।
2. पार्ी िे मुख्य रूप से जमींर्दारों िे खििाफ पंजाब में
Q.39) 1940 िे मुखिम िीग िे िाहौर सं िल्प िे सं र्दभट भू नमहीि मजर्दूरों िे अनधिारों िा बचार् निया।
में, निम्ननिखित िथिों पर नर्चार िरें : 3. पार्ी िे भारत िे नर्भाजि िी मुखिम िीग िी मां ग
1. उसिी िुछ मां गों में से एि थी मुसिमािों िे निए िा समथटि निया।
एि अिग राष्ट्र िे रूप में 'पानिस्ताि' िा निमाट ण। ऊपर नर्दए गए िथिों में से िौि सा/से सही है /हैं ?
2. इस प्रस्तार् िा मसौर्दा नसिंर्दर हयात िाि िे तै यार a) िेर्ि 1 और 3
निया था। b) िेर्ि 3
ऊपर नर्दए गए िथिों में से िौि सा/से सही है /हैं ? c) िेर्ि 2
a) िेर्ि 1 d) उपरोक्त में से िोई िही ं
b) िेर्ि 2
c) 1 और 2 र्दोिों Q.43) एि अिग राष्ट्र िे रूप में पानिस्ताि िे निमाट ण
d) ि तो 1 और ि ही 2 िी मां ग िे इनतहास िे सं बंध में निम्ननिखित िथिों पर
नर्चार िरें
Q.40) 'आयु ष्माि भारत निनजर्ि नमिि (ABDM)' िे 1. चौधरी रहमत अिी िे एि गोिमेज सम्मेिि िे र्दौराि
सं बंध में निम्ननिखित िथिों पर नर्चार िरें : एि अिग राष्ट्र पानिस्ताि िी मां ग िरते हुए अंग्रेजों
1. िेंद्र सरिार द्वारा र्दी गई सखििी िा िाभ उठािे िो एि पैम्फिेर् भें र् निया।
र्ािे िागररिों िे निए नमिि में भागीर्दारी अनिर्ायट 2. पानिस्ताि िो एि अिग राष्ट्र िे रूप में मां ग िरिे
है । िी मां ग उर्दूट िनर् मोहम्मर्द इिबाि से जुड़ी हुई है ।
2. नमिि िे तहत मरीजों िो अपिे मोबाइि फोि पर 3. मुहम्मर्द अिी नजन्ना द्वारा प्रत्यक्ष-िारट र्ाई नर्दर्स िी
अपिे मेनििि ररिॉिट ति पहुं चिे िा अर्सर िुरुआत िे एि अिग राष्ट्र िे रूप में पानिस्ताि िी
नमिेगा। मां ग िो बि नर्दया।
3. ABDM िे तहत रोनगयों िे मेनििि ररिॉिट िो ऊपर नर्दए गए िथिों में से िौि से सही हैं ?
िेर्ि एि र्र्षट िी अनधितम अर्नध ति सं ग्रहीत a) िेर्ि 1 और 2
निया जाता है । b) िेर्ि 2 और 3
ऊपर नर्दए गए िथिों में से िौि सा/से सही है /हैं ? c) िेर्ि 1 और 3
a) िेर्ि 1 d) 1, 2 और 3
b) िेर्ि 2
c) िेर्ि 2 और 3
d) 1, 2 और 3

Forum Learning Centre: Delhi - 2nd Floor, IAPL House, 19 Pusa Road, Karol Bagh, New Delhi - 110005 | Patna - 2nd floor, AG Palace, E Boring Canal
Road, Patna, Bihar 800001 | Hyderabad - 1st & 2nd Floor, SM Plaza, RTC X Rd, Indira Park Road, Jawahar Nagar, Hyderabad, Telangana 500020
9821711605 | https://academy.forumias.com | admissions@forumias.academy | helpdesk@forumias.academy
Page 15

SFG 2023 |LEVEL 1 |Test 22 |

Q.44) With reference to the freedom fighter Q.47) Consider the following pairs regarding the
Subhash Chandra Bose, consider the following Newspapers/ Journals and their Founders/
statements: Editors:
1. He established the Forward bloc party in Newspapers/ Founders/
1939 within the congress. Journals Editors
2. He was elected as president of All India 1. Indian Mirror Devendranath
Kisan Sabha in 1930. Tagore
3. He resigned from Civil service in response 2. Sudharak Gopal Ganesh
to the Non-Cooperation movement. Agarkar
Which of the statements given above is/are 3. Voice of India Dadabhai Naoroji
correct? Which of the pairs given above are correct?
a) 1 and 2 only a) 1 and 2 only
b) 1 and 3 only b) 1 and 3 only
c) 3 only c) 2 and 3 only
d) 1, 2 and 3 d) 1, 2 and 3

Q.45) Which among the following statements is Q.48) Consider the following statements with
incorrect about the liquefaction of soil during respect to Gandhi's attitude towards actions of
earthquakes? Indian National Army (INA):
a) Liquefaction potential of sand can affect the 1. Gandhiji condemned Indian National Army’s
collapse of structures during earthquakes. action of taking help from Nazi Germany, to
b) Irregular Grain Shape of the sand increases liberate India from British rule.
the soil liquification. 2. Gandhi praised the Indian National Army for
c) Glass beads-shaped sand with higher its secular nature.
roundness and sphericity are liquefied first 3. Gandhi supported the cause of freedom of
during earthquakes. Indian National Army’s prisoners from
d) Manufactured sand provides stability British.
against liquefaction. Which of the statements given above are
correct?
Q.46) Which of the following statements is/are a) 1 and 2 only
incorrect features of the elections of 1946 in b) 2 and 3 only
British India? c) 1 and 3 only
1. The election campaign of the Congress was d) 1, 2 and 3
strongly anti-Muslim League in character.
2. It was the first election in British India Q.49) With reference to the role of Subhas
fought on the basis of universal adult Chandra Bose in Freedom Struggle of India,
franchise. consider the following statements
Select the correct answer using the code given 1. He declared war against both Britain and the
below: United States of America (USA).
a) 1 only 2. He secured a promise from the Japanese
b) 2 only government that Japan had no intention of
c) Both 1 and 2 occupying India.
d) Neither 1 nor 2 3. He set up the headquarter of Indian National
Army in Andaman and Nicobar Islands.
Which of the statements given above are
correct?
a) 1 and 2 only
b) 2 and 3 only
c) 1 and 3 only
d) 1, 2 and 3

Forum Learning Centre: Delhi - 2nd Floor, IAPL House, 19 Pusa Road, Karol Bagh, New Delhi - 110005 | Patna - 2nd floor, AG Palace, E Boring Canal
Road, Patna, Bihar 800001 | Hyderabad - 1st & 2nd Floor, SM Plaza, RTC X Rd, Indira Park Road, Jawahar Nagar, Hyderabad, Telangana 500020
9821711605 | https://academy.forumias.com | admissions@forumias.academy | helpdesk@forumias.academy
Page 16

SFG 2023 |LEVEL 1 |Test 22 |

Q.44) स्वतं त्रता से िािी सु भार्ष चंद्र बोस िे सं र्दभट में Q.47) समाचार पत्रों/पनत्रिाओं और उििे
निम्ननिखित िथिों पर नर्चार िीनजएः सं स्थापिों/सं पार्दिों िे सं बंध में निम्ननिखित यु ग्ों पर
1. उन्ोंिे 1939 में िां ग्रेस िे भीतर फॉरर्िट ब्लॉि पार्ी नर्चार िरें :
िी स्थापिा िी। समाचार उिके संस्थापक/संपादक
2. 1930 में उन्ें अखिि भारतीय निसाि सभा िा पत्र/पधत्रकाएं
अध्यक्ष चु िा गया। 1. इं नियि नमरर र्दे र्ेंद्रिाथ र्ै गोर
3. उन्ोंिे असहयोग आं र्दोिि िे जर्ाब में नसनर्ि से र्ा 2.सु धारि गोपाि गणे ि आगरिर
से इस्तीफा र्दे नर्दया। 3. र्ॉइस ऑफ इं निया र्दार्दाभाई िौरोजी
ऊपर नर्दए गए िथिों में से िौि सा/से सही है /हैं ? ऊपर नर्दए गए यु ग्ों में से िौि-से सही हैं ?
a) िेर्ि 1 और 2 a) िेर्ि 1 और 2
b) िेर्ि 1 और 3 b) िेर्ि 1 और 3
c) िेर्ि 3 c) िेर्ि 2 और 3
d) 1, 2 और 3 d) 1, 2 और 3

Q.45) भू िंप िे र्दौराि नमट्टी िे द्रर्ीिरण िे बारे में Q.48) आज़ार्द नहन्द फौज (आईएिए) िे िायों िे प्रनत
निम्ननिखित में से िौि सा िथि गित है ? गां धी िे दृनष्ट्िोण िे सं बंध में निम्ननिखित िथिों पर
a) भू िंप िे र्दौराि रे त िी द्रर्ीिरण क्षमता सं रचिाओं नर्चार िरें :
िे िहिे िो प्रभानर्त िर सिती है । 1. गां धीजी िे भारत िो निनर्ि िासि से मुक्त िरािे िे
b) रे त िे अनियनमत र्दािे िे आिार से नमट्टी िा निए िाजी जमट िी से मर्दर्द िेिे िी आज़ार्द नहन्द फौज
द्रर्ीिरण बढ़ जाता है । िी िारट र्ाई िी निंर्दा िी।
c) उच्च गोिाई और गोिाई िे साथ िां च िे मोनतयों िे 2. गां धी िे भारतीय राष्ट्रीय से िा िी धमटनिरपेक्ष प्रिृनत िे
आिार िी रे त िो भू िंप िे र्दौराि सबसे पहिे द्रनर्त निए उसिी प्रिंसा िी।
निया जाता है । 3. गां धी िे अंग्रेजों से इं नियि िेििि आमी िे िैनर्दयों
d) निनमटत रे त द्रर्ीिरण िे खििाफ खस्थरता प्रर्दाि िरती िी आजार्दी िे िारण िा समथटि निया।
है । ऊपर नर्दए गए िथिों में से िौि से सही हैं ?
a) िेर्ि 1 और 2
Q.46) निम्ननिखित में से िौि सा/से िथि निनर्ि भारत b) िेर्ि 2 और 3
में 1946 िे चुिार्ों िी नर्िेर्षताओं िे सम्बन्ध में गित c) िेर्ि 1 और 3
है /हैं ? d) 1, 2 और 3
1. िां ग्रेस िा चुिार् प्रचार िा स्वरूप पूरी तरह से
मुखिम िीग नर्रोधी था। Q.49) भारत िे स्वतं त्रता सं ग्राम में सु भार्ष चंद्र बोस िी
2. यह निनर्ि भारत में सार्ट भौनमि र्यस्क मतानधिार भू नमिा िे सं र्दभट में , निम्ननिखित िथिों पर नर्चार िरें
िे आधार पर िड़ा गया पहिा चुिार् था। 1. उसिे निर्े ि और सं युक्त राज्य अमेररिा (यू एसए)
िीचे नर्दए गए िूर् िा प्रयोग िर सही उत्तर चुनिए: र्दोिों िे खििाफ यु द्ध िी घोर्षणा िी।
a) िेर्ि 1 2. उन्ोंिे जापाि सरिार से एि र्ार्दा हानसि निया नि
b) िेर्ि 2 जापाि िा भारत पर िब्जा िरिे िा िोई इरार्दा िही ं
c) 1 और 2 र्दोिों है ।
d) ि तो 1 और ि ही 2 3. उन्ोंिे अंिमाि और नििोबार द्वीप समूह में आज़ार्द
नहन्द फौज िा मुख्यािय स्थानपत निया।
ऊपर नर्दए गए िथिों में से िौि से सही हैं ?
a) िेर्ि 1 और 2
b) िेर्ि 2 और 3
c) िेर्ि 1 और 3
d) 1, 2 और 3

Forum Learning Centre: Delhi - 2nd Floor, IAPL House, 19 Pusa Road, Karol Bagh, New Delhi - 110005 | Patna - 2nd floor, AG Palace, E Boring Canal
Road, Patna, Bihar 800001 | Hyderabad - 1st & 2nd Floor, SM Plaza, RTC X Rd, Indira Park Road, Jawahar Nagar, Hyderabad, Telangana 500020
9821711605 | https://academy.forumias.com | admissions@forumias.academy | helpdesk@forumias.academy
Page 17

SFG 2023 |LEVEL 1 |Test 22 |

Q.50) Consider the following statements


regarding Tiger Conservation:
1. Tiger has been classified as “Endangered”
under IUCN Red List.
2. Government of India has launched Project
Tiger in 1973, to comply with St. Petersburg
Declaration on Tiger Conservation.
3. India achieved the goal of doubling the tiger
numbers in 2018, four years in advance from
the targeted year 2022.
Which of the statements given above are
correct?
a) 1 and 2 only
b) 2 and 3 only
c) 1 and 3 only
d) 1, 2 and 3

Forum Learning Centre: Delhi - 2nd Floor, IAPL House, 19 Pusa Road, Karol Bagh, New Delhi - 110005 | Patna - 2nd floor, AG Palace, E Boring Canal
Road, Patna, Bihar 800001 | Hyderabad - 1st & 2nd Floor, SM Plaza, RTC X Rd, Indira Park Road, Jawahar Nagar, Hyderabad, Telangana 500020
9821711605 | https://academy.forumias.com | admissions@forumias.academy | helpdesk@forumias.academy
Page 18

SFG 2023 |LEVEL 1 |Test 22 |

Q.50) बाघ सं रक्षण िे सं बंध में निम्ननिखित िथिों पर


नर्चार िरें :
1. र्ाइगर िो IUCN रे ि निस्ट् िे तहत "िुप्तप्राय" िे
रूप में र्गीिृत निया गया है ।
2. बाघ सं रक्षण पर सें र् पीर्सट बगट घोर्षणा िे अिुपािि
िे निए भारत सरिार िे 1973 में प्रोजेक्ट र्ाइगर
िॉन्च निया था।
3. भारत िे िनक्षत र्र्षट 2022 से चार साि पहिे 2018 में
बाघों िी सं ख्या िो र्दोगु िा िरिे िा िक्ष्य हानसि
निया।
ऊपर नर्दए गए िथिों में से िौि से सही हैं ?
a) िेर्ि 1 और 2
b) िेर्ि 2 और 3
c) िेर्ि 1 और 3
d) 1, 2 और 3

Forum Learning Centre: Delhi - 2nd Floor, IAPL House, 19 Pusa Road, Karol Bagh, New Delhi - 110005 | Patna - 2nd floor, AG Palace, E Boring Canal
Road, Patna, Bihar 800001 | Hyderabad - 1st & 2nd Floor, SM Plaza, RTC X Rd, Indira Park Road, Jawahar Nagar, Hyderabad, Telangana 500020
9821711605 | https://academy.forumias.com | admissions@forumias.academy | helpdesk@forumias.academy
Page 1 of 33

SFG 2023 | LEVEL 1 | Test #22 – Solutions |


Q.1) Which of the following led to the introduction of English Education in India?
1. Charter Act of 1813
2. General Committee of Public Instruction, 1823
3. Orientalist and Anglicist Controversy
Select the correct answer using the code given below:
a) 1 and 2 only
b) 2 only
c) 1 and 3 only
d) 1, 2 and 3

Ans) d
Exp) Option d is the correct answer.
Option 1 is correct: The Charter Act of 1813 required the company to spend rupees 1 lakh annually for
encouraging learned Indians and promoting the knowledge of modern sciences in India.
Option 2 is correct: In 1823, the Governor -General -in Council appointed a “General Committee of Public
Instruction”, which had the responsibility to grant the one lakh of rupees for education.
Option 3 is correct: Orientalist -Anglicists Controversy: Within the General Committee on Public
Instruction, the Anglicists argued that the government spending on education should be exclusively for
modern studies. The Orientalists said while western sciences and literature should be taught to prepare
students to take up jobs, emphasis should be placed on expansion of traditional Indian learning.
Source: UPSC CSE Pre 2018

Q.2) Consider the following statements with respect to the Indian National Army (INA) trials:
1. Congress adopted a resolution in the Bombay session in support of the Indian National Army’s cause.
2. Prem Kumar Sehgal, Shah Nawaz khan and Gurbaksh Singh Dhillon were the first INA prisoners to face
the trial.
3. Following the public protest, British reduced the death sentence of INA prisoners to deportation for life.
Which of the statements given above are correct?
a) 1 and 2 only
b) 2 and 3 only
c) 1 and 3 only
d) 1, 2 and 3

Ans) a
Exp) Option a is the correct answer.
The Indian National Army trials also known as the INA trials and the Red Fort trials was the British Indian
trial of a number of officers of the Indian National Army (INA) between 1945 and 1946, on various charges
of treason, torture, murder and abetment to murder. The INA prisoners were the ones who fought with
Japanese troops during World War II in Burma.
Statement 1 is correct: At the first post-War Congress session in September 1945 at Bombay, a strong
resolution was adopted declaring Congress support for the INA cause. Those who supported the INA cause
in varying degrees, apart from the Congress, included the Muslim League, Communist Party, Unionists,
Akalis, Justice Party, Ahrars in Rawalpindi, Rashtriya Swayamsevak Sangh, Hindu Mahasabha and the Sikh
League.
Statement 2 is correct: British held the first trial at the Red Fort in Delhi in November 1945 and putting on
dock together a Hindu, Prem Kumar Sehgal, a Muslim, Shah Nawaz Khan, and a Sikh, Gurbaksh Singh
Dhillon. These three prisoners were the first to face the trials.

Forum Learning Centre: Delhi - 2nd Floor, IAPL House, 19 Pusa Road, Karol Bagh, New Delhi - 110005 | Patna - 2nd floor, AG Palace, E Boring Canal Road,
Patna, Bihar 800001 | Hyderabad - 1st & 2nd Floor, SM Plaza, RTC X Rd, Indira Park Road, Jawahar Nagar, Hyderabad, Telangana 500020
9821711605 | https://academy.forumias.com | admissions@forumias.academy | helpdesk@forumias.academy
Page 2 of 33

SFG 2023 | LEVEL 1 | Test #22 – Solutions |


Statement 3 is incorrect: The British never sentenced any prisoners to death. The actual punishment for
Shah Nawaz Khan, Gurubaksh Singh Dhillon and Prem Sahgal were to be dismissed from service and
handed transportation for life. However, this punishment too was remitted later.
Source: https://theprint.in/india/governance/remembering-the-red-fort-trials-that-tipped-india-
towards-complete-freedom/145260/

Q.3) With reference to the Royal Indian Navy (RIN) revolt, consider the following statements:
1. Only the members of the armed forces participated in the revolt.
2. It demanded withdrawal of Indian troops from Indonesia.
3. Congress gave its official support to the revolt.
4. Communists and socialist played a major role in guiding the mutiny.
Which of the statements given above are correct?
a) 1 and 4 only
b) 2 and 3 only
c) 2 and 4 only
d) 2, 3 and 4 only

Ans) c
Exp) Option c is the correct answer.
Statement 1 is incorrect: Royal Indian Navy revolt is not limited to armed forces alone. Students had
boycotted classes, strikes and processions were organised to express sympathy with the students and the
ratings and to condemn official repression.
Statement 2 is correct: The demands of RIN revolt included the release of INA (Indian National Army)
personnel and other political prisoners; withdrawal of Indian troops from Indonesia; and the acceptance
of Indian officers only as superiors.
Statement 3 is incorrect: While the congress lauded the spirit of the people it did not officially support
these struggles as it felt their tactics and timing were wrong. It was evident for congress that the
Government would be able to repress.
Statement 4 is correct: Royal Indian Navy agitation was led majorly by the communists, socialist and
forward blocists. The congress role was majorly diffusing the revolutionary situation prompted by the
fear that the situation would go out of its control or by the concern that disciplined armed forces were vital
in free India that the party would soon rule.
Source: India’s struggle for Independence: Post war National Upsurge

Q.4) Consider the following statements regarding Wardha Scheme of Basic Education:
1. The detailed national scheme for basic education was formulated by Zakir Hussain committee.
2. It advocated a free and compulsory nationwide education system through mother tongue for the first
seven years of schooling.
3. The scheme could not be put into practice due to the start of the First World War.
Which of the statements given above is/are correct?
a) 1 only
b) 2 only
c) 1 and 2 only
d) 1, 2 and 3

Ans) c
Exp) Option c is the correct answer.

Forum Learning Centre: Delhi - 2nd Floor, IAPL House, 19 Pusa Road, Karol Bagh, New Delhi - 110005 | Patna - 2nd floor, AG Palace, E Boring Canal Road,
Patna, Bihar 800001 | Hyderabad - 1st & 2nd Floor, SM Plaza, RTC X Rd, Indira Park Road, Jawahar Nagar, Hyderabad, Telangana 500020
9821711605 | https://academy.forumias.com | admissions@forumias.academy | helpdesk@forumias.academy
Page 3 of 33

SFG 2023 | LEVEL 1 | Test #22 – Solutions |


Statement 1 is correct: The Congress had organised a National Conference on Education in October 1937
in Wardha. In the light of the resolutions passed there, Zakir Hussain committee formulated a detailed
national scheme for basic education.
Statement 2 is correct: The scheme had the provisions like inclusion of a basic handicraft in the syllabus;
first seven years of schooling to be an integral part of a free and compulsory nationwide education
system (through mother tongue).
Statement 3 is incorrect: There was not much development of the Wardha Scheme of Basic Education,
because of the start of the Second World War and the resignation of the Congress ministries (October
1939).
Source: A Brief History of Modern India Revised and Enlarged Edition 2019 Chapter 30 Development of
Education

Q.5) Consider the following statements with reference to the Acropora corals, that has been seen recently
in news:
1. They are the fast-growing corals found in Australia’s Great Barrier Reef.
2. They do not contain zooxanthellae and derive nourishment from capturing different forms of plankton.
3. They can strongly withstand environmental pressures such as rising temperatures, cyclones, and
pollution.
Which of the statements given above is/are correct?
a) 1 and 2 only
b) 3 only
c) 1 only
d) 1, 2 and 3

Ans) c
Exp) Option c is the correct answer.
Recently, as per a report Australia's northern and central Great Barrier Reef (GBR) has experienced high
levels of coral reef cover over the past 36 years which has been fueled by the fast-growing Acropora corals
in the region. Acropora Reef is a species of coral found over a large range in the Indian Ocean, the Red
Sea, the Persian Gulf, the Gulf of Aden, New Caledonia, and Fiji.
Statement 1 is correct: Acropora species of the Corals are some of the major reef corals responsible for
building the immense calcium carbonate substructure in the Great Barrier Reef. It is a dominant type of
Corals found in the Great Barrier Reef. These corals are fast-growing in nature.
Statement 2 is incorrect: Acropora are common types of corals that live in symbiotic relationship with the
zooxanthellae algae which help the coral survive by providing it with food resulting from photosynthesis.
So, this statement is incorrect.
Statement 3 is incorrect: Incidentally, these fast-growing corals are also the most susceptible to
environmental pressures such as rising temperatures, cyclones, pollution, crown-of-thorn starfish
(COTs) attacks which prey on hard corals and so on. So, the statement is incorrect.
Source: https://www.thehindu.com/sci-tech/energy-and-environment/explained-the-great-barrier-
reefs-recovery-and-vulnerability-to-climate-threats/article65741674.ece
https://www.fisheries.noaa.gov/species/acropora-pharaonis-coral
https://core.ac.uk/download/pdf/303717223.pdf

Q.6) With reference to the Sapru Committee of 1945, consider the following statements:
1. The Committee rejected the Muslim league’s demand for Pakistan.
2. It operated under the mandate of Indian National Congress.

Forum Learning Centre: Delhi - 2nd Floor, IAPL House, 19 Pusa Road, Karol Bagh, New Delhi - 110005 | Patna - 2nd floor, AG Palace, E Boring Canal Road,
Patna, Bihar 800001 | Hyderabad - 1st & 2nd Floor, SM Plaza, RTC X Rd, Indira Park Road, Jawahar Nagar, Hyderabad, Telangana 500020
9821711605 | https://academy.forumias.com | admissions@forumias.academy | helpdesk@forumias.academy
Page 4 of 33

SFG 2023 | LEVEL 1 | Test #22 – Solutions |


3. The Committee recommended the separate electorate for Muslims.
Which of the statements given above is/are correct?
a) 1 and 3 only
b) 1 and 2 only
c) 2 only
d) 1 only

Ans) d
Exp) Option d is the correct answer.
The constitutional proposals of the Sapru committee were commonly referred to as Sapru Committee
Report which was published in 1945.
Statement 1 is correct. The Report rejected the Muslim league’s demand for Pakistan as it was not
convinced that a separate state would be beneficial to any community and division of India will endanger
the peace and growth of the country.
Statement 2 is incorrect. The Committee consisted of 30 members who are experts of public affairs and
did not operate under the mandate of any political party.
Statement 3 is incorrect. The Committee rejected the separate electorate for Muslims and proposed joint
elections with reservation of seats.
Source:
https://www.constitutionofindia.net/historical_constitutions/sapru_committee_report__sir_tej_bah
adur_sapru__1945__1st%20December%201945

Q.7) With reference to the Indian provincial elections of 1946, consider the following statements:
1. The Congress failed to get majority in any of the provinces which were being claimed by the Muslim
League as part of a separate nation i.e. Pakistan.
2. The Muslim league got the majority of Muslim votes.
3. A Unionist-Muslim League-Akali coalition under Khizr Hayat Khan assumed power in Punjab. Which of
the statements given above is/are correct?
a) 1 only
b) 1 and 3 only
c) 2 only
d) 2 and 3 only

Ans) c
Exp) Option c is the correct answer.
In July 1945, Labour Party formed the government in Britain. Clement Attlee took over as the new Prime
Minister and Pethick Lawrence as the new secretary of state for India. In August 1945, elections to central
and provincial assemblies were announced.
Performance of the Congress
(a) It got 91 per cent of non-Muslim votes. It captured 57 out of 102 seats in the Central Assembly.
(b) In the provincial elections, it got a majority in most provinces except in Bengal, Sindh and Punjab.
(c) The Congress majority provinces included the North West Frontier Province (NWFP) and Assam which
were being claimed for Pakistan. Hence, statement 1 is not correct.
Performance of the Muslim League
1) It got 86.6 per cent of the Muslim votes.
2) It captured the 30 reserved seats in the Central Assembly. In the provincial elections, it got a majority in
Bengal and Sindh.

Forum Learning Centre: Delhi - 2nd Floor, IAPL House, 19 Pusa Road, Karol Bagh, New Delhi - 110005 | Patna - 2nd floor, AG Palace, E Boring Canal Road,
Patna, Bihar 800001 | Hyderabad - 1st & 2nd Floor, SM Plaza, RTC X Rd, Indira Park Road, Jawahar Nagar, Hyderabad, Telangana 500020
9821711605 | https://academy.forumias.com | admissions@forumias.academy | helpdesk@forumias.academy
Page 5 of 33

SFG 2023 | LEVEL 1 | Test #22 – Solutions |


3) Unlike in 1937, now the League clearly established itself as the dominant party among Muslims. Hence,
statement 2 is correct.
4)In Punjab, a Unionist-Congress-Akali coalition under Khizr Hayat Khan assumed power. Hence,
statement 3 is incorrect.
Source: Spectrum Modern India 2019-20 Edition

Q.8) Consider the following statements regarding the various acts on press regulation:
1. In Vernacular Press Act (1878), the magistrate’s action was final, and no appeal could
be made in a court of law.
2. In Indian Press Act (1910), the local government was authorized to forfeit an offending newspaper.
Which of the statements given above is/are correct?
a) 1 only
b) 2 only
c) Both 1 and 2
d) Neither 1 nor 2

Ans) c
Exp) Option c is the correct answer.
Statement 1 is correct: The Vernacular Press Act (1878) was designed to better control the vernacular press
and effectively punish and repress seditious writing. The Act included the following provisions. The district
magistrate was empowered to call upon the printer/publisher of any vernacular newspaper to enter into a
bond with the government undertaking not to cause disaffection against the government. The printer and
publisher could also be required to deposit security which could be forfeited if the regulation were
contravened, and press equipment could be seized if the offence re-occurred. The magistrate’s action was
final, and no appeal could be made in a court of law. A vernacular newspaper could get exemption from
the operation of the Act by submitting proofs to a government censor.
Statement 2 is correct: This Indian Press Act (1910), revived the worst features of Vernacular Press Act
(1878). The local government was empowered to demand a security at registration from the
printer/publisher. It was also authorized to forfeit an offending newspaper. The printer of a newspaper
was required to submit two copies of each issue to local government free of charge.
Knowledge Base: The Vernacular Press Act (1878) came to be nicknamed “the gagging Act”. The worst
features of this Act were discrimination between English and Vernacular press, and no right of appeal.
Under VPA, proceedings were instituted against Som Prakash, Bharat Mihir, Dacca Prakash and Samachar.
Incidentally, the Amrita Bazar Patrika turned overnight into an English newspaper to escape the VPA.
Source: https://ncert.nic.in/textbook/pdf/jess305.pdf
Ch 29, Spectrum

Q.9) With reference to C. Rajagopalachari (CR) Formula, consider the following statements:
1. It asked Muslim league to form a provisional government with the Indian National Congress.
2. It proposed a plebiscite in the North-West and North-East India for creation of a separate nation.
3. Mohammed Ali Jinnah opposed the plan as he wanted the Congress to accept the Two Nation Theory.
4. Mahatma Gandhi condemned the CR Plan as it indirectly accepted the League’s demand for separate
nation.
Which of the statements given above are correct?
a) 1, 2 and 3 only
b) 2, 3 and 4 only
c) 1, 2 and 4 only
d) 1, 3 and 4 only

Forum Learning Centre: Delhi - 2nd Floor, IAPL House, 19 Pusa Road, Karol Bagh, New Delhi - 110005 | Patna - 2nd floor, AG Palace, E Boring Canal Road,
Patna, Bihar 800001 | Hyderabad - 1st & 2nd Floor, SM Plaza, RTC X Rd, Indira Park Road, Jawahar Nagar, Hyderabad, Telangana 500020
9821711605 | https://academy.forumias.com | admissions@forumias.academy | helpdesk@forumias.academy
Page 6 of 33

SFG 2023 | LEVEL 1 | Test #22 – Solutions |


Ans) a
Exp) Option a is the correct answer.
During Indian national Movement (INM), Various efforts were made to solve the constitutional crisis. In
fact, some individuals also tried to come up with constitutional proposals. For example, C. Rajagopalachari
(CR), the veteran Congress leader, prepared a formula for Congress-League cooperation in 1944. The main
points of the CR Plan and the objections raised against it included as follows:
Statement 1 is correct. CR plan asked Indian Muslim League (IML) to endorse Congress demand for
independence and also asked Indian Muslim League (IML) to cooperate with Congress in forming a
provisional government at centre.
Statement 2 is correct. CR Plan tacitly accepted the League’s demand for Pakistan. It mentioned that after
the end of the war, the entire population of Muslim majority areas in the North-West and North-East
India to decide by a plebiscite, whether or not to form a separate sovereign state. In case of acceptance of
partition, agreement to be made jointly for safeguarding defence, commerce, communications, etc.
Statement 3 is correct. Jinnah opposed the plan. First, he wanted the Indian National Congress (INC) to
accept the two-nation theory. Secondly, he wanted only the Muslims of North-West and North-East to
vote in the plebiscite and not the entire population. Thirdly, he also opposed the idea of a common centre.
Statement 4 is incorrect. Mahatma Gandhi supported the C. Rajagopalachari formula. However, some
sections of nationalist leaders opposed it. For example, Vir Savarkar, a Hindu leader condemned the CR
Plan.
Source: Brief History of India, Quit India Movement Chapter, Page 454

Q.10) The CAPSTONE, recently seen in news, is a/an:


a) protein that can be used for genetic engineering.
b) nano satellite launched by NASA.
c) important geological feature of Jurassic period.
d) medicine that can be used to treat Ebola virus.

Ans) b
Exp) Option b is the correct answer.
Recently, NASA has launched Cislunar Autonomous Positioning System Technology Operations and
Navigation Experiment, CAPSTONE, a microwave oven-sized CubeSat weighing just 25 kg. As a pathfinder
for Gateway, a Moon-orbiting outpost that is part of NASA’s Artemis program, CAPSTONE will help reduce
risk for future spacecraft by validating innovative navigation technologies and verifying the dynamics of
this halo-shaped orbit.
Source: https://www.nasa.gov/directorates/spacetech/small_spacecraft/capstone/

Q.11) Regarding Wood's Dispatch, which of the following statements are true?
1. Grants-in-Aid system was introduced.
2. Establishment of universities was recommended.
3. English as a medium of instruction at all levels of education was recommended.
Select the correct answer using the code given below:
a) 1 and 2 only
b) 2 and 3 only
c) 1 and 3 only
d) 1, 2 and 3

Ans) a

Forum Learning Centre: Delhi - 2nd Floor, IAPL House, 19 Pusa Road, Karol Bagh, New Delhi - 110005 | Patna - 2nd floor, AG Palace, E Boring Canal Road,
Patna, Bihar 800001 | Hyderabad - 1st & 2nd Floor, SM Plaza, RTC X Rd, Indira Park Road, Jawahar Nagar, Hyderabad, Telangana 500020
9821711605 | https://academy.forumias.com | admissions@forumias.academy | helpdesk@forumias.academy
Page 7 of 33

SFG 2023 | LEVEL 1 | Test #22 – Solutions |


Exp) Option a is the correct answer.
Option 1 and 2 are correct: Wood's Dispatch recommended setting up of universities at Madras, Calcutta
and Bombay. It recommended a system of grants-in-aid to encourage private enterprise.
Option 3 is incorrect: Wood's Dispatch recommended promotion of both Western educations at the same
time of promotion of Vernacular education at primary level for education of masses. It is often termed as
Magna Carta of India education.
Source: UPSC CSE Pre 2018

Q.12) Which of the following was/were the proposals of the Cabinet Mission plan?
1. It proposed the partition of India into two nations, India and Pakistan.
2. The Princely states would no longer be under the paramountcy of the British Government.
3. It had proposed a two-tier executive and legislature at provincial and union levels.
Select the correct answer using the code given below:
a) 1 and 2 only
b) 1 and 3 only
c) 2 and 3 only
d) 2 only

Ans) d
Exp) Option d is the correct answer.
In March 1946, the British Cabinet sent a three-member Cabinet Mission to India. Its members were Pethick
Lawrence, Stafford Cripps and A.V. Alexander. Its aim was to negotiate the setting up of a national
government and set into motion a machinery for transfer of power.
Statement 1 is incorrect. Cabinet Mission suggested that India should remain united (and not supported
2 nations) and constitute itself as a loose three-tier confederation with some autonomy for Muslim-
majority areas. It was to have a weak central government controlling only foreign affairs, defence and
communications with the existing provincial assemblies
Statement 2 is correct: Provinces were to have full autonomy and residual powers. Princely states were
no longer to be under paramountcy of the British government. They would be free to enter into an
arrangement with successor governments or the British Government.
Statement 3 is incorrect: Cabinet mission of three-member committee had recommended the three-tier
executive and legislature at provincial, section and union levels.
Source: Page 389, https://ncert.nic.in/ncerts/l/lehs305.pdf
Page 430, https://ncert.nic.in/textbook/pdf/lehs306.pdf
https://ncert.nic.in/ncerts/l/hess205.pdf
Freedom Struggle, Bipin Chandra

Q.13) Consider the following statements with respect to the Mountbatten plan:
1. Under the plan, referendum was to be conducted in the North West Frontier Province and Sylhet district of Bengal.
2. Under the plan, a boundary commission was to be set up if partition was to be effected.
3. Under the plan, there was provision for two Constituent Assemblies if partition was to be affected.
4. The Plan was opposed by both Congress and Muslim League.
Which of the statements given above are correct?
a) 1 and 2 only
b) 2, 3 and 4 only
c) 1, 2 and 3 only
d) 1, 2, 3 and 4

Forum Learning Centre: Delhi - 2nd Floor, IAPL House, 19 Pusa Road, Karol Bagh, New Delhi - 110005 | Patna - 2nd floor, AG Palace, E Boring Canal Road,
Patna, Bihar 800001 | Hyderabad - 1st & 2nd Floor, SM Plaza, RTC X Rd, Indira Park Road, Jawahar Nagar, Hyderabad, Telangana 500020
9821711605 | https://academy.forumias.com | admissions@forumias.academy | helpdesk@forumias.academy
Page 8 of 33

SFG 2023 | LEVEL 1 | Test #22 – Solutions |


Ans) c
Exp) Option c is the correct answer.
Mountbatten plan was announced on June 3, 1947. It recommended for
• Punjab and Bengal Legislative Assemblies would meet in two groups, Hindus and Muslims, to vote for
partition. If a simple majority of either group voted for partition, then these provinces would be partitioned.
• Referendums in North West Frontier Province and Sylhet district of Bengal would be conducted. This
referendum was to decide the fate of these areas. Hence, statement 1 is correct.
• In case of partition, two dominions and two constituent assemblies would be created. Therefore, provision
of separate constituent assembly for Pakistan is correct. Hence, statement 3 is correct.
• Sindh would take its own decision.
• Accession of Hyderabad to Pakistan ruled out.
• A boundary commission to be set up if partition was to be effected. Hence, statement 2 is correct.
Both Congress and Muslim league accepted the proposals of Mountbatten plan. Hence, statement 4 is
incorrect.
Source: https://egyankosh.ac.in/bitstream/123456789/44324/3/Unit-24.pdf
Pg 493, ch 25, Spectrum

Q.14) Consider the following statements regarding India Independence Act of 1947:
1. The Constituent Assembly of India which also worked as the provisional parliament enacted the India Independence Act,
1947.
2. Under the act, both the Dominions of India and Pakistan, will have Governor Generals appointed by
Britain’s King/Queen.
3. The office of the Secretary of State for India will be continued till the adoption of the Indian constitution.
Which of the statements given above is/are correct?
a) 2 only
b) 1 only
c) 1 and 3 only
d) 1 and 2 only

Ans) a
Exp) Option a is the correct answer.
Statement 1 is incorrect: On July 5, 1947 the British Parliament passed the Indian Independence Act which was
based on the Mountbatten Plan, and the Act got royal assent on July 18, 1947. The Act was implemented on August 15,
1947.
Statement 2 is correct: Both the dominions (India and Pakistan) were to have Governor Generals
appointed by Britain’s King/Queen to represent him for the purposes of the Governments of the
dominion. The Act also provided for one common General if both the dominions of India and Pakistan,
agreed.
Statement 3 is incorrect: The office of the Secretary of State for India and his advisors was abolished
and affairs relating to the Dominion of India and Pakistan were to be conducted in future by the Secretary
of the Commonwealth Relations Department.
Knowledge Base: The features of India Independence act, 1947 are as follows:
• The Act provided for the creation of two independent Dominions, namely, India and Pakistan from 1st
August, 1947.
• The Act defined the territories of the two Dominions adjustable after the award of boundary Commission.
• The Indian States were free to accede to either of the two new Dominions.

Forum Learning Centre: Delhi - 2nd Floor, IAPL House, 19 Pusa Road, Karol Bagh, New Delhi - 110005 | Patna - 2nd floor, AG Palace, E Boring Canal Road,
Patna, Bihar 800001 | Hyderabad - 1st & 2nd Floor, SM Plaza, RTC X Rd, Indira Park Road, Jawahar Nagar, Hyderabad, Telangana 500020
9821711605 | https://academy.forumias.com | admissions@forumias.academy | helpdesk@forumias.academy
Page 9 of 33

SFG 2023 | LEVEL 1 | Test #22 – Solutions |


• The Legislature of each dominion was empowered to frame the laws for the governance of the Dominion.
No law made by that Dominion will be treated as null and void on the ground that it conflicted with any
laws of England.
• Those persons who had been appointed by the Secretary of State or Secretary of State-in-Council to Civil
Service and the Crown of India before August 15, 1947 would continue in this service after independence
Source: Pg 10, https://egyankosh.ac.in/bitstream/123456789/43772/1/Unit-5.pdf
https://ncert.nic.in/ncerts/l/lehs305.pdf

Q.15) Which of the following best describes the DeFi (Decentralized Financing) system, that has been
recently seen in news?
a) a system that provides a range of financial services using digital assets.
b) a technology used for the mining of crypto currencies.
c) a new loan disbursal scheme launched by the RBI for achieving financial inclusion.
d) a new type of Non-Banking Financial Company set up to provide credit facilities in rural and tribal areas.

Ans) a
Exp) Option a is the correct answer.
DeFi refers to Decentralised Financing which is a component of the Web 3.0 concept. DeFi provides a range
of financial services like trading in securities, lending & borrowing, paying for services/ assets, or
maintaining saving accounts using digital assets such as cryptocurrencies using a peer-to-peer system.
In case of DeFi system entire financial transaction occurs between two ordinary individuals without the
involvement of a central authority like a tech company or a central bank or a commercial bank which
generally regulates financial transactions currently.
These transactions will use the Distributed Ledger Technology (DLT) (such as blockchain) for carrying out
these transactions. DLT means that the resources used to complete the transaction and record the financial
agreement will be distributed across several personal systems and not concentrated in servers/ clouds
owned by a few tech corporates or government.
Source: https://indianexpress.com/article/explained/explained-crypto-banking-and-decentralized-
finance-7489689/
https://indianexpress.com/article/technology/crypto/what-is-web3-and-why-it-matters-7659054/
https://www.businessworld.in/article/How-Defi-And-Web-3-0-Could-Shape-The-Future-Of-
Finance/17-06-2022-433058/
https://www.forbes.com/advisor/investing/cryptocurrency/defi-decentralized-finance/
https://economictimes.indiatimes.com/tech/trendspotting/explained-how-defi-could-one-day-
liberate-finance/articleshow/87511218.cms?from=mdr
https://www.nytimes.com/interactive/2022/03/18/technology/what-is-defi-cryptocurrency.html

Q.16) Consider the following statements regarding the Boundary commissions set up to draw borders of
India and Pakistan after partition:
1. Sir Cyril John Radcliffe was made the Chairman of two boundary commissions of Punjab and Bengal.
2. The boundary commissions included two nominees each of the Indian National Congress and Muslim
League respectively.
3. The award of the Boundary Commissions was announced before India declared its independence.
Which of the statements given above is/are correct?
a) 1 and 2 only
b) 2 only
c) 1 and 3 only
d) 2 and 3 only

Forum Learning Centre: Delhi - 2nd Floor, IAPL House, 19 Pusa Road, Karol Bagh, New Delhi - 110005 | Patna - 2nd floor, AG Palace, E Boring Canal Road,
Patna, Bihar 800001 | Hyderabad - 1st & 2nd Floor, SM Plaza, RTC X Rd, Indira Park Road, Jawahar Nagar, Hyderabad, Telangana 500020
9821711605 | https://academy.forumias.com | admissions@forumias.academy | helpdesk@forumias.academy
Page 10 of 33

SFG 2023 | LEVEL 1 | Test #22 – Solutions |


Ans) a
Exp) Option a is the correct answer.
Statement 1 is correct: In June 1947, Sir Cyril John Radcliffe, a British lawyer, was made the Chairman
of two boundary commissions of Punjab and Bengal and given the task to draw up the new borders of
India and Pakistan.
Statement 2 is correct: The boundary commissions of Punjab and Bengal included two nominees each of
the Indian National Congress and Muslim League respectively. The Punjab commission had Justice Mehr
Chand Mahajan, Justice Teja Singh, Justice Din Mohammad and Justice Muhammad Munir as members. The
Bengal commission comprised Justice CC Biswas, Justice BK Mukherjee, Justice Abu Saleh Akram and
Justice SA Rehman.
Statement 3 is incorrect: On August 17, 1947, the award of the Boundary Commissions for the partition
of Punjab and Bengal was announced. It means the award was announced after the Independence.
Source: https://indianexpress.com/article/explained/1947-boundary-commission-awards-punjab-
bengal-india-8086687/

Q.17) She was the first woman to contest for a legislative seat in India, in the Madras provincial elections.
She joined Indian National Congress in 1927. During the Salt March to Dandi, she convinced Gandhi to give
equal opportunity to women to be in the forefront of the march. Later, she joined Seva Dal and trained
women activists.
Which of the following personality is mentioned above?
a) Kamaladevi Chattopadhyay
b) Sarojini Naidu
c) Vijaya Lakshmi Pandit
d) Kamala Nehru

Ans) a
Exp) Option a is the correct answer.
Option a is correct: Kamaladevi Chattopadhyay is one of the unsung freedom fighters. Kamaladevi was
born on April 3, 1903, in Mangalore. She was the first woman to run for a legislative seat in India, in the
Madras provincial elections. Many of the iconic cultural institutions in India today exist because of her
vision, these include the National School of Drama, Sangeet Natak Akademi, etc. She joined Indian National
Congress in 1927. During the Salt March to Dandi, she convinced Gandhi to give women equal
opportunity to be in the forefront of the March. Later, she joined Seva Dal and trained women activists.
Option b is incorrect: Sarojini Naidu was born in Hyderabad on 13 February, 1879. She was educated in
Madras, London and Cambridge. She was arrested for participating in 1930 Salt March. She was appointed
the President of the INC in 1925 and later became the Governor of the United Provinces in 1947, becoming
the first woman to hold the office of Governor in the Dominion of India.
Option c is incorrect: Vijaya Lakshmi Pandit was born as Swarup Kumari Nehru on 18 August 1900 to the
Nehru family. She was imprisoned by the British on three different occasions, in 1932-1933, 1940, and 1942-
1943. In 1936 she was elected to the Assembly of the United Provinces, and in 1937 became minister of local
self-government and public health, the first Indian woman ever to become a cabinet minister.
Option d is incorrect: Kamala Nehru was born on August 1st, 1899. She was home schooled. The Non-
Cooperation Movement of 1921 saw Kamala Nehru's entry into the freedom struggle of the country. She
organised no-tax campaigns along with other women volunteers including Durgabai and Kamaladevi
Chattopadhyaya.
Source: https://www.indiatoday.in/education-today/gk-current-affairs/story/why-google-
remembers-kamaladevi-chattopadhyay-the-unsung-feminist-freedom-fighter-1203511-2018-04-03

Forum Learning Centre: Delhi - 2nd Floor, IAPL House, 19 Pusa Road, Karol Bagh, New Delhi - 110005 | Patna - 2nd floor, AG Palace, E Boring Canal Road,
Patna, Bihar 800001 | Hyderabad - 1st & 2nd Floor, SM Plaza, RTC X Rd, Indira Park Road, Jawahar Nagar, Hyderabad, Telangana 500020
9821711605 | https://academy.forumias.com | admissions@forumias.academy | helpdesk@forumias.academy
Page 11 of 33

SFG 2023 | LEVEL 1 | Test #22 – Solutions |


https://amritmahotsav.nic.in/unsung-heroes-detail.htm?319
https://www.indiatoday.in/education-today/gk-current-affairs/story/kamala-nehru-119th-birth-
anniversary-1302267-2018-08-01
https://www.constitutionofindia.net/constituent_assembly_members/vijaya_lakshmi_pandit

Q.18) Consider the following pairs regarding the Acts on development of Indian Press:
Act related to Provision
press
1. Censorship Every newspaper
of Press Act, should print the
1799 names of the
printers.
2. Licensing Replaced Metcalfe’s
Act, 1857 Act of 1835
3. Registration Copy of book to be
Act, 1867 submitted to the local
government
How many pairs given above is/are correct?
a) Only one pair
b) Only two pairs
c) All three pairs
d) None of the pairs

Ans) b
Exp) Option b is the correct answer.
Pair 1 is correct: Lord Wellesley enacted the Censorship of Press Act (1799), anticipating the French
invasion of India. It imposed wartime press restrictions including pre-censorship. These restrictions were
relaxed under Lord Hastings, who had progressive views, and in 1818, pre-censorship was dispensed with.
As per this act, every newspaper should print the names of the printers, editor and proprietor.
Pair 2 is incorrect: Licensing Act, 1857 did not replaced Metcalfe’s Act of 1835. Registration Act, 1867
replaced the Metcalfe’s Act of 1835. Due to the emergency caused by the 1857 revolt, Licensing Act, 1857
was enacted. It imposed licensing restrictions in addition to the already existing registration procedure
laid down by Metcalfe Act and the government reserved the right to stop publication and circulation of any
book, newspaper or printed matter.
Pair 3 is correct: Registration Act ,1867 replaced the Metcalfe’s Act of 1835 and was of a regulatory, not
restrictive, nature. As per the Act, every book/ newspaper was required to print the name of the printer
and the publisher and the place of the publication; and a copy was to be submitted to the local government
within one month of the publication of a book.
Knowledge Base: Pg 557, ch 29, Spectrum

Q.19) With reference to Subhash Chandra Bose and Indian National Army (INA), consider the following
statements:
1. Subhash Chandra Bose opposed the Quit India Movement as yet another ineffective nonviolent struggle.
2. The defeat of British by INA during Imphal campaign encouraged its members to liberate India from
British.

Forum Learning Centre: Delhi - 2nd Floor, IAPL House, 19 Pusa Road, Karol Bagh, New Delhi - 110005 | Patna - 2nd floor, AG Palace, E Boring Canal Road,
Patna, Bihar 800001 | Hyderabad - 1st & 2nd Floor, SM Plaza, RTC X Rd, Indira Park Road, Jawahar Nagar, Hyderabad, Telangana 500020
9821711605 | https://academy.forumias.com | admissions@forumias.academy | helpdesk@forumias.academy
Page 12 of 33

SFG 2023 | LEVEL 1 | Test #22 – Solutions |


Which of the statements given above is/are correct?
a) 1 only
b) 2 only
c) Both 1 and 2
d) Neither 1 nor 2

Ans) d
Exp) Option d is the correct answer.
Statement 1 is incorrect: Subhas Chandra Bose considered the Indian National Army movement to be
never opposed to but rather complementary to the Quit India Movement. He described the Quit India
Movement as “India’s epic struggle” and kept the outside world informed about all events in India and
tried to secure sympathy and help for India’s struggle for freedom.
Statement 2 is incorrect: Imphal Campaign was a failure. One INA battalion commanded by Shah Nawaz
was allowed to accompany the Japanese army to the Indo-Burma front and participate in the Imphal
campaign. But the discriminatory treatment meted out to Indians which included being denied rations,
arms and being made to do menial work for the Japanese army, completely demoralised the members. The
failure of the Imphal campaign and the following arrest of INA members by the British culminated in the
famous INA trial in India.
Source: India’s Struggle for Independence: The Quit India movement and the INA

Q.20) Which one of the statements given below most appropriately describes the term ‘Gene Therapy’?
a) It is a type of medical test that is used in identification of changes in genes, chromosomes, or proteins.
b) It is a technique of enhancing capabilities of an organism beyond the normal levels through genetic
modifications.
c) It is the ability of genetic improvement through the correction of altered genes or site-specific
modifications.
d) It is a technique that mainly aims at the prevention of communicable diseases through DNA analysis.

Ans) c
Exp) Option c is the correct answer.
Gene therapy is a technique that modifies a person’s genes to treat or cure disease. It is understood as the
ability of genetic improvement through the correction of altered genes or site-specific modifications.
Gene therapies can work by several mechanisms:
 Replacing a disease-causing gene with a healthy copy of the gene
 Inactivating a disease-causing gene that is not functioning properly
 Introducing a new or modified gene into the body to help treat a disease
Gene therapy products are being studied to treat diseases including cancer, genetic diseases, and
infectious diseases.

Forum Learning Centre: Delhi - 2nd Floor, IAPL House, 19 Pusa Road, Karol Bagh, New Delhi - 110005 | Patna - 2nd floor, AG Palace, E Boring Canal Road,
Patna, Bihar 800001 | Hyderabad - 1st & 2nd Floor, SM Plaza, RTC X Rd, Indira Park Road, Jawahar Nagar, Hyderabad, Telangana 500020
9821711605 | https://academy.forumias.com | admissions@forumias.academy | helpdesk@forumias.academy
Page 13 of 33

SFG 2023 | LEVEL 1 | Test #22 – Solutions |

There are a variety of types of gene therapy products, including:


1) Plasmid DNA: Circular DNA molecules can be genetically engineered to carry therapeutic genes into
human cells.
2) Viral vectors: Viruses have a natural ability to deliver genetic material into cells, and therefore some
gene therapy products are derived from viruses. Once viruses have been modified to remove their ability
to cause infectious disease, these modified viruses can be used as vectors (vehicles) to carry therapeutic
genes into human cells.
3) Bacterial vectors: Bacteria can be modified to prevent them from causing infectious disease and then
used as vectors (vehicles) to carry therapeutic genes into human tissues.
4) Human gene editing technology: The goals of gene editing are to disrupt harmful genes or to repair
mutated genes.
5) Patient-derived cellular gene therapy products: Cells are removed from the patient, genetically
modified (often using a viral vector) and then returned to the patient.
Option a is incorrect: Genetic testing is a type of medical test that identifies changes in genes,
chromosomes, or proteins. The results of a genetic test can confirm or rule out a suspected genetic
condition or help determine a person's chance of developing or passing on a genetic disorder.
Option b is incorrect: Gene therapy seeks to alter genes to correct genetic defects and thus prevent or
cure genetic diseases. Whereas, Genetic engineering aims to modify the genes to enhance the capabilities
of the organism beyond what is normal.
Option d is incorrect: Gene therapy mainly deals with genetic diseases which are categorized as non-
communicable diseases.
Source: https://medicine.missouri.edu/centers-institutes-labs/health-ethics/faq/gene-
therapy#:~:text=The%20distinction%20between%20the%20two,organism%20beyond%20what%20is%2
0normal.
What is Gene Therapy? | FDA
Gene therapy - Mayo Clinic
Genetic screening - PMC (nih.gov)

Q.21) The Vital-Vidhvansak, the first monthly journal to have the untouchable people as its target audience
was published by
a) Gopal Baba Walangkar
b) Jyotiba Phule

Forum Learning Centre: Delhi - 2nd Floor, IAPL House, 19 Pusa Road, Karol Bagh, New Delhi - 110005 | Patna - 2nd floor, AG Palace, E Boring Canal Road,
Patna, Bihar 800001 | Hyderabad - 1st & 2nd Floor, SM Plaza, RTC X Rd, Indira Park Road, Jawahar Nagar, Hyderabad, Telangana 500020
9821711605 | https://academy.forumias.com | admissions@forumias.academy | helpdesk@forumias.academy
Page 14 of 33

SFG 2023 | LEVEL 1 | Test #22 – Solutions |


c) Mohandas Karamchand Gandhi
d) Bhimrao Ramji Ambedkar

Ans) a
Exp) Option a is the correct answer.
Gopal Baba Walangkar, also known as Gopal Krishna, (1840-1900) was an early proponent to release the
untouchable people of India from their historic socio-economic oppression, and is generally considered to
be the pioneer of that movement. He developed a racial theory to explain the oppression and also published
the first journal targeted at the untouchable people. Vital-Vidhvansak means (Destroyer of Brahmanical
or Ceremonial Pollution), which was the first to have the untouchable people as its target audience.
Source: UPSC CSE 2020

Q.22) T B Macaulay's Minute on Indian Education of 1835 is regarded by many as the most significant
document in the history of Indian education. In this context, which of the following statements is incorrect
about this Macaulay’s Minute?
a) The minute rejected indigenous literatures and languages and promoted English.
b) The minute encapsulates the concept of the White man's burden.
c) The minute was based on the orientalist view on education.
d) The minute aimed to help in achieving the Civilizing mission of the British Government.

Ans) c
Exp) Option c is the correct answer.
The Macaulay’s Minute is seen as a watershed that not only governed the educational policy of the British
administration after 1835 but also reflected a crucial change in the strategy of control and subjugation of
the native population
Statement c is incorrect: Macaulay’s Minute is based on Anglicist view on education. Macaulay attacks
the Orientalists, accusing them of bribing the Indian student to learn Native languages and literatures.
Macaulay finds the Native vernaculars so poor in their vocabulary that no useful instruction can be given
in these languages.
Statement a, b and d are correct: According to minute, the limited government resources were to be
devoted to teaching of Western sciences and literature through the medium of English language alone. It
rejected indigenous literatures and languages. The Minute also encapsulates the concept of the White
man's burden, which means that it is the responsibility of enlightened Europe to inculcate modern values
among the less civilised communities. The new native who will emerge after receiving English education
will be "Indian in blood and colour but English in taste, in opinions, in morals and in intellect. In this way,
the Minute will help in achieving the Civilizing mission of British Government.
Knowledge Base: Civilizing mission was a set of ideas and practices that was used by the colonial powers
to legitimize the establishment and continuation of overseas colonies. According to the British, the main
aim of civilizing mission was to morally and materially ‘uplifting’ and ‘developing’ the supposedly ‘backward’
or ‘rude’ people of India to make them more civilized and more modern.
Source: https://egyankosh.ac.in/bitstream/123456789/23158/1/Unit-2.pdf

Q.23) Which of the following actions/policies by the British were responsible for the creation of Pakistan
as a separate nation?
1. The Cripps mission’s proposal provided legitimacy to the demand for the creation of a separate nation.
2. Recognition of Muslim league as a sole representative of Muslims in India by Wavell’s plan.

Forum Learning Centre: Delhi - 2nd Floor, IAPL House, 19 Pusa Road, Karol Bagh, New Delhi - 110005 | Patna - 2nd floor, AG Palace, E Boring Canal Road,
Patna, Bihar 800001 | Hyderabad - 1st & 2nd Floor, SM Plaza, RTC X Rd, Indira Park Road, Jawahar Nagar, Hyderabad, Telangana 500020
9821711605 | https://academy.forumias.com | admissions@forumias.academy | helpdesk@forumias.academy
Page 15 of 33

SFG 2023 | LEVEL 1 | Test #22 – Solutions |


3. The British invited the Congress to form interim government without taking into consideration the
Muslim League’s demands.
4. Acceptance of Pakistan as a separate nation by the Cabinet mission plan.
Select the correct answer using the code given below:
a) 1 and 2 only
b) 1 and 3 only
c) 2 and 3 only
d) 1, 3 and 4 only

Ans) b
Exp) Option b is the correct answer.
Statement 1 is correct: Cripps Mission’s proposals gave a fillip to the activities of the Muslim League and
provided legitimacy to the Pakistan demand by accommodating it in their provision for provincial
autonomy. In fact, there was a provision in the mission that any province unwilling to join the Indian
dominion could form a separate union and have a separate constitution. Hence, at a time when the demand
had hardly been taken seriously by Indians, its sympathetic consideration by officialdom was a great
service to the cause of Pakistan.
Statement 2 is incorrect: Wavell plan had no such proposal. The idea of Wavell plan was to reconstruct
the governor-general’s executive council (except the governor-general and the commander-in-chief, all
members of the executive council were to be Indians). For this purpose, a Simla conference was convened
by the viceroy, Lord Wavell in June 1945. In this conference Jinnah insisted that league alone has the right
to nominate Muslims to the executive council, but congress was opposed to it. Thus, not able to settle
their differences Simla conference broke down without taking any decision.
Statement 3 is correct: The British invited the Congress to form an Interim Government which came
into being on 2nd September 1946 with Jawaharlal Nehru functioning as its de facto head despite of the
objections of Muslim League let by Muhamad Ali Jinnah. In response to this Jinnah launched direct action
day in Calcutta on August 16, 1946, with a slogan “We will fight and get Pakistan”. It gave a fillip to the
demand for creation of Pakistan as a separate nation.
Statement 4 is incorrect: The Cabinet Mission Plan 1946 rejected the demand for a full-fledged separate
nation of Pakistan. The Cabinet Mission did so as it would not solve the communal problem and it found
no reason to merge the non-Muslim parts of Bengal, Punjab, and Assam. According to the mission It would
also result in the disruption of the Indian armed forces, telegraph lines, and the postal system. It would also
cause a dilemma among the Princely States to join India or Pakistan. In place of a separate state of Pakistan,
the Cabinet mission suggested a plan to safeguard the interests of the Muslim minority within the overall
framework of unity of the country.
Source: 12th NCERT Volume 3 Chapter-Understanding Partition
https://egyankosh.ac.in/bitstream/123456789/20097/1/Unit-36.pdf

Q.24) Consider the following statements regarding the development of Education under the British rule in
India:
1. Under the 1913 Resolution on Education policy, the British government agreed to provide free primary education to all.
2. In 1902, Raleigh Commission was set up to go into conditions and prospects of universities in India.
3. The Sergeant plan of 1944 provided for technical, commercial and arts education.
Which of the statements given above are correct?
a) 1 and 2 only
b) 1 and 3 only
c) 2 and 3 only
d) 1, 2 and 3

Forum Learning Centre: Delhi - 2nd Floor, IAPL House, 19 Pusa Road, Karol Bagh, New Delhi - 110005 | Patna - 2nd floor, AG Palace, E Boring Canal Road,
Patna, Bihar 800001 | Hyderabad - 1st & 2nd Floor, SM Plaza, RTC X Rd, Indira Park Road, Jawahar Nagar, Hyderabad, Telangana 500020
9821711605 | https://academy.forumias.com | admissions@forumias.academy | helpdesk@forumias.academy
Page 16 of 33

SFG 2023 | LEVEL 1 | Test #22 – Solutions |


Ans) c
Exp) Option c is the correct answer.
Statement 1 is incorrect: In 1906, the progressive state of Baroda introduced compulsory primary education throughout
its territories. National leaders urged the government to do so for British India. In its 1913 Resolution on Education Policy,
the government refused to take up the responsibility of compulsory education, but accepted the policy of removal of
illiteracy and urged provincial governments to take early steps to provide free elementary education to the poorer and more
backward sections.
Statement 2 is correct: In 1902, Raleigh Commission was set up to go into conditions and prospects of universities
in India and to suggest measures for improvement in their constitution and working. Based on its recommendations, the
Indian Universities Act was passed in 1904.
Statement 3 is correct: The Sergeant Plan was worked out by the Central Advisory Board of Education in 1944. It
recommended for adequate technical, commercial and arts education; pre-primary education for 3-6 years age group and
free universal and compulsory elementary education for 6-11 years age group; abolition of intermediate course; liquidation
of adult illiteracy in 20 years.
Knowledge Base: The Indian Universities Act was passed in 1904. As per the Act
1) universities were to give more attention to study and research
2) The number of fellows of a university and their period in office were reduced and most fellows were to
be nominated by the Government.
3) Government was to have powers to veto universities’ senate regulations and could amend these
regulations or pass regulations on its own.
4) Conditions were to be made stricter for affiliation of private colleges; and
5) Five lakh rupees were to be sanctioned per annum for five years for improvement of higher education
and universities
Source: Pg 567, ch 30, Spectrum

Q.25) With reference to the Panchayats Extension to The Scheduled Areas (PESA) Act of 1996, consider the
following statements:
1. The act aims to extend the Constitutional provisions of Panchayats to Fifth and Sixth scheduled areas.
2. Chhattisgarh was the first state in India to implement the provisions of PESA act.
3. Rules regarding the implementation of the act can be framed by the Centre alone and not by the states.
Which of the above statements is/are incorrect?
a) 1 and 3 only
b) 1 only
c) 2 and 3 only
d) 1, 2 and 3

Ans) d
Exp) Option d is the correct answer.
Statement 1 is incorrect: The PESA Act was enacted in 1996 “to provide for the extension of the provisions
of Part IX of the Constitution relating to the Panchayats to the Scheduled Areas”. Under the PESA Act,
Scheduled Areas are those referred to in Article 244(1), which says that the provisions of the Fifth
Schedule (not the sixth schedule) shall apply to the Scheduled Areas and Scheduled Tribes in states other
than Assam, Meghalaya, Tripura, and Mizoram. It ensures self-governance through Gram Sabhas (village
assemblies) for people living in the Scheduled Areas.
Statement 2 is incorrect: With PESA rules being notified recently, Chhattisgarh became the seventh state
in the country to frame rules and implement PESA after Andhra Pradesh, Gujarat, Himachal Pradesh,
Maharashtra, Rajasthan and Telangana.

Forum Learning Centre: Delhi - 2nd Floor, IAPL House, 19 Pusa Road, Karol Bagh, New Delhi - 110005 | Patna - 2nd floor, AG Palace, E Boring Canal Road,
Patna, Bihar 800001 | Hyderabad - 1st & 2nd Floor, SM Plaza, RTC X Rd, Indira Park Road, Jawahar Nagar, Hyderabad, Telangana 500020
9821711605 | https://academy.forumias.com | admissions@forumias.academy | helpdesk@forumias.academy
Page 17 of 33

SFG 2023 | LEVEL 1 | Test #22 – Solutions |


Statement 3 is incorrect: Recently, Chhattisgarh notified the rules for implementation of the Panchayats
(Extension to the Scheduled Areas) (PESA) Act, 1996 on the occasion of World Tribal Day. Basically, PESA is
a Central law that has been in existence since the 1990s. However, its implementation is State-specific.
Chhattisgarh has formed its set of rules while many States with significant tribal population are yet to
implement the provisions of the Act. (Hence, rules for the implementation of PESA act are framed by the
state governments.)
Source: Chhattisgarh govt. implements PESA Rule-2022 - The Hindu
https://pib.gov.in/PressReleaseIframePage.aspx?PRID=1794826

Q.26) Which of the following statements is correct with reference to the provisions of the Wavell’s
Breakdown plan?
a) Except governor- general and commander-in-chief, all other members of the executive council to be
Indians.
b) Transfer of power to separate provinces and an option given to remain independent.
c) Punjab and Bengal legislative assemblies would meet in two groups of Hindus and Muslims to vote for
the partition of their provinces.
d) Withdrawal of British army and British officials to the Muslim provinces of north west and north east of
India.

Ans) d
Exp) Option d is the correct answer.
Wavell presented his plan to the Cabinet Mission in May 1946. It was considered a balanced approach
between “repression” and “scuttle”.
Option a is incorrect: Except governor- general and commander-in-chief, all members of executive
council to be kept Indians provision was given under Wavell plan. It was aimed at reconstructing the
governor-general’s executive council. It provided that with exception of governor-general and
commander-in-chief, all members were to be Indians. Governor-general was also given veto to be
exercised with advice of his ministers.
Option b is incorrect: The plan Balkan was introduced in 1947. It was aimed at transferring the power to
separate provinces or to a confederation, if formed before the transfer. Punjab and Bengal were given the
option to vote for partition of their provinces. The various units thus formed along with the princely states
would have the option of joining India or Pakistan or remaining separate. The plan was quickly abandoned.
Option c is incorrect: The Mountbatten Plan provided this provision. Punjab and Bengal Legislative
Assemblies to meet in two groups of Hindus and Muslims for voting their partition. If a simple majority of
either group voted for partition, then these provinces would be partitioned.
Option d is correct: Breakdown plan proposed the withdrawal of the British Army and officials to the
Muslim provinces of North-West and North-East India. The rest of the country would be hand over to
the Congress. Wavell’s plan was evidence of:
1) British recognition of the impossibility of suppressing any future Congress-led rebellion.
2) desire in some high official circles to make a “Northern Ireland” of Pakistan.
Source: Spectrum, chapter on post war national scenario

Q.27) The National Defence Council was set up as a result of which of the following events?
a) Indian National Session of Haripura
b) Cripps Mission
c) August Offer
d) Indian National Session of Faizpur

Forum Learning Centre: Delhi - 2nd Floor, IAPL House, 19 Pusa Road, Karol Bagh, New Delhi - 110005 | Patna - 2nd floor, AG Palace, E Boring Canal Road,
Patna, Bihar 800001 | Hyderabad - 1st & 2nd Floor, SM Plaza, RTC X Rd, Indira Park Road, Jawahar Nagar, Hyderabad, Telangana 500020
9821711605 | https://academy.forumias.com | admissions@forumias.academy | helpdesk@forumias.academy
Page 18 of 33

SFG 2023 | LEVEL 1 | Test #22 – Solutions |


Ans) c
Exp) Option c is the correct answer.
On 8 August 1940, the Viceroy Lord Linlithgow made a proposal called the August Offer which expanded
the Executive Council to include more Indians. The National Defence Council was set up with purely
advisory functions.
The National Defence Council’s main purpose was to bring the war effort in the Provinces and States, as
well as in the ranks of commerce, industry and labour, into more effective touch with the Central
Government.
Source: A Brief History of Modern India by Rajiv Ahir 2019 Edition - Chapter 22
Nationalist Response in the Wake of World War II P 439, 440.
https://api.parliament.uk/historic-hansard/commons/1941/sep/11/viceroys-executive-council-and-
national

Q.28) With reference to interim government of 1946, consider the following statements:
1. Muslim league members joined the interim government with the condition of not giving up with their
‘direct action’ program.
2. It functioned according to the provision of Government of India Act of 1919.
3. Jawaharlal Nehru was the President of the Viceroys Executive Council.
Which of the statements given above is/are correct?
a) 1 and 2 only
b) 2 and 3 only
c) 1 only
d) 1, 2 and 3

Ans) a
Exp) Option a is the correct answer.
On, September 1946 interim government was constituted as pure congress government. Wavell later
brought the Muslim League into the Interim Government on October 26, 1946.
Statement 1 is correct: Muslim league decided to join the interim government without surrendering
‘direct action’. Earlier, the interim government consisted only congress members. On October 26, 1946 the
League was allowed to join:
1) without giving up the ‘direct action’
2) despite its rejection of the long-term and short-term plans put forward by cabinet mission.
3) And despite its insistence on compulsory grouping and decisions being taken by a majority vote given
by section of population for the whole region.
Statement 2 is correct: It functioned under the provision of government of India act 1919. A federal
scheme provided under the Government of India Act of 1935, was not implemented due to the opposition
from India’s princely states. As a result, the interim government functioned according to the older
Government of India Act of 1919.
Statement 3 is incorrect: The members of the interim Government were members of
the Viceroy’s Executive Council. The Viceroy continued to be the head of the Council. But, Jawaharlal Nehru
was designated as the Vice-President of the Council.
Source: Spectrum, chapter on post war national scenario
https://indianexpress.com/article/explained/september-2-when-indias-interim-govt-was-formed-in-
1946-5959889/

Forum Learning Centre: Delhi - 2nd Floor, IAPL House, 19 Pusa Road, Karol Bagh, New Delhi - 110005 | Patna - 2nd floor, AG Palace, E Boring Canal Road,
Patna, Bihar 800001 | Hyderabad - 1st & 2nd Floor, SM Plaza, RTC X Rd, Indira Park Road, Jawahar Nagar, Hyderabad, Telangana 500020
9821711605 | https://academy.forumias.com | admissions@forumias.academy | helpdesk@forumias.academy
Page 19 of 33

SFG 2023 | LEVEL 1 | Test #22 – Solutions |


Q.29) What was the immediate reason leading to Muhammad Ali Jinnah’s declaration of 16 August, 1946 as
Direct Action Day?
a) Rejection of demand to form full-fledged Pakistan under Cabinet Mission plan.
b) Nehru’s statement that the constituent assembly would not have the compulsory grouping of provincial
assemblies.
c) Congress formed the interim government without accepting the demands put forward by Muslim league.
d) Overthrowing the coalition government in Punjab.

Ans) b
Exp) Option b is the correct answer.
Jinnah gave call for ‘direct action’ from August 16, 1946. It was aimed at achieving Pakistan. Jinnah declared,
“Today we have said goodbye to constitutions and constitutional methods.”
Option a is incorrect: Although demand for full-fledged Pakistan was rejected under cabinet mission plan.
(Though this could be one of the reasons leading to direct action day but not the immediate reason). Also,
the long-term provision of cabinet mission was accepted by both Muslim league as well as congress. The
long-term provision includes:
• After the first general elections, a province would be free to come out of a group and after 10 years, a
province would be free to call for a reconsideration of the group or the union constitution.
• Meanwhile, an interim government would be formed from the constituent assembly.
Option b is correct: Jinnah called for direct action after the Nehru’s statement. On July 10, 1946 Nehru
gave statement in AICC meeting “We are not bound by a single thing except that we have decided to go
into the Constituent Assembly”. This implied that the Constituent Assembly to be formed would be
sovereign and decide its rules of procedure. Hence, there would be no compulsory grouping (as demanded
by Muslim league). As a result on July 29, 1946 the League withdrew its acceptance of the long-term plan in
response to Nehru’s statement and gave call for direct action.
Option c is incorrect: The Interim Government was formed on 2nd September 1946 while the day decided
for direct action was 16 august 1946. Also, interim government was formed with Congress members alone
with Nehru as de facto head. Wavell later on brought the Muslim League into the Interim Government on
October 26, 1946.
Option d is incorrect: On February 20, 1947 Attlee’s Statement consists of the provision of transfer of power
to more than one centre. It was accepted by Congress but the league encouraged with this, decided to
launch a civil disobedience movement. It was aimed at overthrowing the coalition government in Punjab.
It was not related to Direct Action Day.
Source: Spectrum, chapter on post war national scenario, pg no 494

Q.30) Consider the following pair of places and their location:


Place Location
1. Al-Aqsa Mosque Turkey
2. Katchal Island Sri Lanka
3. Lake Garda Italy
Which of the above pairs is/are correctly matched?
a) 2 only
b) 3 only
c) 1 and 2 only
d) 1, 2 and 3

Ans) b
Exp) Option b is the correct answer.

Forum Learning Centre: Delhi - 2nd Floor, IAPL House, 19 Pusa Road, Karol Bagh, New Delhi - 110005 | Patna - 2nd floor, AG Palace, E Boring Canal Road,
Patna, Bihar 800001 | Hyderabad - 1st & 2nd Floor, SM Plaza, RTC X Rd, Indira Park Road, Jawahar Nagar, Hyderabad, Telangana 500020
9821711605 | https://academy.forumias.com | admissions@forumias.academy | helpdesk@forumias.academy
Page 20 of 33

SFG 2023 | LEVEL 1 | Test #22 – Solutions |


Pair 1 is incorrect: Al-Aqsa Mosque is a congregational mosque in the Old City of Jerusalem. The Al-Aqsa
Mosque compound has been the most sensitive site in the Israel-Palestine conflict. The Al-Aqsa Mosque
compound is referred to as Temple Mount by Jews. It is a holy site for both Islam and Judaism.

Pair 2 is incorrect: Katchal island is a part of India’s Nicobar archipelago. NASA has recently highlighted
the loss of mangrove cover on Katchal island. More than 90% of mangrove cover on the island was lost due
to the Indian Ocean Tsunami.

Forum Learning Centre: Delhi - 2nd Floor, IAPL House, 19 Pusa Road, Karol Bagh, New Delhi - 110005 | Patna - 2nd floor, AG Palace, E Boring Canal Road,
Patna, Bihar 800001 | Hyderabad - 1st & 2nd Floor, SM Plaza, RTC X Rd, Indira Park Road, Jawahar Nagar, Hyderabad, Telangana 500020
9821711605 | https://academy.forumias.com | admissions@forumias.academy | helpdesk@forumias.academy
Page 21 of 33

SFG 2023 | LEVEL 1 | Test #22 – Solutions |


Pair 3 is correct: Lake Garda is located in Italy. It is the largest lake in the country. Italy has recently
undergone its worst drought and that has reduced the Lake Garda to its lowest level ever.

Source: https://www.usnews.com/news/world/articles/2022-08-13/italys-lake-garda-shrinks-to-
near-historic-low-amid-drought#
https://www.downtoearth.org.in/news/environment/nasa-highlights-loss-of-mangrove-cover-on-
katchal-island-in-the-nicobars-84236
https://www.thehindu.com/news/international/watch-why-is-the-al-aqsa-mosque-compound-a-
flashpoint-for-the-israel-palestine-
conflict/article65367369.ece#:~:text=In%20the%201967%20Arab%2DIsraeli,violent%20clashes%20over
%20the%20years.

Q.31) With reference to Indian History, the Members of the Constituent Assembly from the Provinces were:
a) directly elected by the people of those Provinces
b) nominated by the Indian National Congress and the Muslim League
c) elected by the Provincial Legislative Assemblies
d) selected by the Government for their expertise in constitutional matters

Ans) c
Exp) Option c is the correct answer.
The members of the Constituent Assembly from the Provinces were to be elected to indirectly by the
members of the individual provincial legislative assemblies.
Source: UPSC CSE Pre 2013

Forum Learning Centre: Delhi - 2nd Floor, IAPL House, 19 Pusa Road, Karol Bagh, New Delhi - 110005 | Patna - 2nd floor, AG Palace, E Boring Canal Road,
Patna, Bihar 800001 | Hyderabad - 1st & 2nd Floor, SM Plaza, RTC X Rd, Indira Park Road, Jawahar Nagar, Hyderabad, Telangana 500020
9821711605 | https://academy.forumias.com | admissions@forumias.academy | helpdesk@forumias.academy
Page 22 of 33

SFG 2023 | LEVEL 1 | Test #22 – Solutions |


Q.32) With reference to Congress’s stand towards Cabinet Mission proposals, consider the following
statements:
1. Congress interpreted Cabinet Mission proposals as mandatory grouping of provincial assemblies into
three sections.
2. Congress all together rejected the proposals under Cabinet mission.
3. Congress objected to the proposal that provinces need to wait till the first general elections to come out
of a group.
Which of the statements given above is/are correct?
a) 1 and 3 only
b) 2 and 3 only
c) 3 only
d) 1 and 2 only

Ans) c
Exp) Option c is the correct answer.
The Cabinet Mission reached Delhi on March 24, 1946. It was constituted to provide solution on the issues:
(1) interim government.
(2) principles and procedures for framing a new constitution giving freedom to India
Statement 1 is incorrect: Each party Muslim league as well as congress interpreted cabinet mission
differently. To the Congress, the Cabinet Mission Plan was interpreted as:
1) The grouping of existing provincial assemblies’ intro three section was optional. Hence, rejecting the
plan of Pakistan.
2) one constituent assembly was envisaged; and the League no longer had a veto
Statement 2 is incorrect: Congress as well as Muslim league accepted the long-term plan put forward
by the Cabinet mission. The long-term plan includes:
1) After the first general elections, a province would be free to come out of a group and after 10 years, a
province would be free to call for a reconsideration of the group or the union constitution.
2) Meanwhile, an interim government would be formed from the constituent assembly
Statement 3 is correct. Congress objected to the plan that provinces need to wait till first general
elections to come out of a group. They proposed that provinces should have the option of not joining a
group in the first place.
Source: Spectrum, chapter on post war national scenario, pg no 493

Q.33) With reference to the interim Government of 1946, consider the following statements:
1. It was formed from the Constituent Assembly which was elected in August 1946.
2. Dr Babasaheb Ambedkar headed the portfolio of Law in the Interim government.
3. It appointed a committee to advise the government on nationalizing the armed forces.
Which of the above statements is/are correct?
a) 1 and 3 only
b) 2 and 3 only
c) 2 only
d) 1 and 2 only

Ans) a
Exp) Option a is the correct answer.
On 2nd September 1946, the Interim Government of India was formed to oversee the transition of the
country from a British colony to an independent republic.

Forum Learning Centre: Delhi - 2nd Floor, IAPL House, 19 Pusa Road, Karol Bagh, New Delhi - 110005 | Patna - 2nd floor, AG Palace, E Boring Canal Road,
Patna, Bihar 800001 | Hyderabad - 1st & 2nd Floor, SM Plaza, RTC X Rd, Indira Park Road, Jawahar Nagar, Hyderabad, Telangana 500020
9821711605 | https://academy.forumias.com | admissions@forumias.academy | helpdesk@forumias.academy
Page 23 of 33

SFG 2023 | LEVEL 1 | Test #22 – Solutions |


Statement 1 is correct: The Interim Government was formed from the Constituent Assembly which was
elected in August 1946.The election to the Constituent Assembly was not direct and representatives were
elected by the provincial assemblies.
Statement 2 is incorrect. Dr. Babasaheb Ambedkar was not part of the interim government formed in 1946.
Jogendra Nath Mandal of Muslim League headed the portfolio of Law in interim government.
Statement 3 is correct: In November 1946, India ratified the Convention on International Civil Aviation. In
the same month, a committee was appointed to advise the government on nationalizing the armed forces.
Knowledge Base: The Interim Government was formed as a provisional government between an imperial
structure and a democratic structure.
1) It lasted till 15th August 1947 when India became independent and was partitioned into India and
Pakistan.
2) In these elections, the Indian National Congress (INC) won around 69% of the seats and had a majority.
The Congress Party won 208 seats and the Muslim League won 73 seats.
3) In the Interim Government, the Viceroy’s Executive Council was equivalent to the position of the
Council of Ministers acted as the executive.
4) Even though the Muslim League refused to be a part of the Interim Government, insisting on their
demand for a separate nation, it eventually became a part of it. In Muhammad Ali Jinnah’s words, the
League was “going into the Interim Government to get a foothold to fight for… the cherished goal of
Pakistan.
Source: https://indianexpress.com/article/explained/september-2-when-indias-interim-govt-was-
formed-in-1946-5959889/
https://ncert.nic.in/textbook/pdf/lehs306.pdf

Q.34) The primary objective of the Pirpur Committee was to:


a) prepare report on the atrocities supposedly committed by the Congress ministries formed after 1937
elections.
b) resolve issues pertaining to minorities that has affected Indian political discourse.
c) examine the governance reforms proposed by the Government of India Act, 1919.
d) enquire into conditions of universities of India.

Ans) a
Exp) Option a is the correct answer.
Pirpur Committee in 1938 was formed by the All-India Muslim League who were annoyed with the
Congress for not sharing power with them. The objective of the committee was to prepare a detailed report
on the atrocities supposedly committed by the Congress ministries. In its report the committee charged
the Congress with interference in the religious rites, suppression of Urdu in favour of Hindi, denial of
proper representation and of the oppression of Muslims in the economic sphere.
Source: Modern History, Spectrum, Chapter-21, Congress Rule in Provinces, Pg. 415

Q.35) Consider the following statements with reference to measures for Data protection in India:
1. CERT-In is the national nodal agency for responding to cybersecurity threats.
2. As per RBI, generally the payment data shall be stored in systems located in India.
3. Ministry of Home Affairs launched ‘Cyber Surakshith Bharat’ initiative to raise awareness about the
cyber-crimes.
Which of the statements given above is/are correct?
a) 1 and 2 only
b) 3 only
c) 1, 2 and 3

Forum Learning Centre: Delhi - 2nd Floor, IAPL House, 19 Pusa Road, Karol Bagh, New Delhi - 110005 | Patna - 2nd floor, AG Palace, E Boring Canal Road,
Patna, Bihar 800001 | Hyderabad - 1st & 2nd Floor, SM Plaza, RTC X Rd, Indira Park Road, Jawahar Nagar, Hyderabad, Telangana 500020
9821711605 | https://academy.forumias.com | admissions@forumias.academy | helpdesk@forumias.academy
Page 24 of 33

SFG 2023 | LEVEL 1 | Test #22 – Solutions |


d) 1 only

Ans) a
Exp) Option a is the correct answer
Data protection is the process of safeguarding important information from corruption, compromise, or
loss.
Statement 1 is correct: Computer Emergency Response Team - India (CERT-In) is an organization of the
Ministry of Electronics and Information Technology with the objective of securing Indian cyberspace.
CERT-In is empowered under Section 70B of the Information Technology Act to collect, analyze and
disseminate information on cyber security incidents. It is the national nodal agency which deals with
cybersecurity threats like hacking and phishing.
Statement 2 is correct: With data localization, RBI aims to protect personal data of the country's citizens
by restricting data on servers outside the country's geographical boundaries. As per the RBI data
localization rules, the entire payment data shall be stored in systems located only in India.
Statement 3 is incorrect: Cyber Surakshith Bharat initiative was conceptualized with the mission to
spread awareness about cyber-crime and build capacities of Chief Information Security Officers (CISOs)
and frontline IT officials, across all government departments. It was launched in 2018 by the Ministry of
Electronics and Information Technology (MeitY) (not the Ministry of Home Affairs).
Source: https://m.rbi.org.in/scripts/FAQView.aspx?Id=130
https://www.cert-in.org.in/
https://www.business-standard.com/article/economy-policy/centre-withdraws-personal-data-
protection-bill-2019-to-present-new-bill-
122080301226_1.html#:~:text=The%20PDP%20bill%20was%20first,personal%20and%20non%2Dpersonal
%20datasets.

Q.36) With reference to the Constituent Assembly of India, consider the following statements:
1. The Indian National Congress officially demanded a Constituent Assembly for the first time in 1935.
2. The demand for the constituent assembly was accepted, in principle, by the British in August Offer.
3. It was constituted under the scheme formulated by Cripps Mission Plan.
Which of the statements given above is/are correct?
a) 1 and 2 only
b) 2 and 3 only
c) 1 only
d) 1, 2, and 3

Ans) a
Exp) Option a is the correct Answer.
The idea of a constituent assembly was put forward for the first time by MN Roy. In 1935, the Indian National
Congress (INC), for the first time, officially called for a constituent assembly to frame a constitution for
India.
Statement 1 is correct: In 1935, the Indian National Congress (INC), for the first time, officially demanded
a Constituent Assembly to frame the Constitution of India. In 1938, Jawaharlal Nehru, on behalf of the INC
declared that ‘the Constitution of free India must be framed, without outside interference, by a Constituent
Assembly elected on the basis of the adult franchise’.
Statement 2 is correct: The demand was finally accepted in principle by the British Government in what is
known as the ‘August Offer’ of 1940. In 1942, Sir Stafford Cripps, a member of the cabinet, came to India

Forum Learning Centre: Delhi - 2nd Floor, IAPL House, 19 Pusa Road, Karol Bagh, New Delhi - 110005 | Patna - 2nd floor, AG Palace, E Boring Canal Road,
Patna, Bihar 800001 | Hyderabad - 1st & 2nd Floor, SM Plaza, RTC X Rd, Indira Park Road, Jawahar Nagar, Hyderabad, Telangana 500020
9821711605 | https://academy.forumias.com | admissions@forumias.academy | helpdesk@forumias.academy
Page 25 of 33

SFG 2023 | LEVEL 1 | Test #22 – Solutions |


with a draft proposal of the British Government on the framing of an independent Constitution to be
adopted after World War II.
Statement 3 is incorrect: The Constituent Assembly was constituted in November 1946 under the scheme
formulated by the Cabinet Mission Plan
Source: https://ncert.nic.in/textbook/pdf/hess301.pdf

Q.37) Which of the following statements are correct with reference to the Objective Resolution introduced
by Jawaharlal Nehru?
1. It laid down the general philosophy behind the Indian Constitution.
2. It promises to provide adequate safeguards for the minorities and backward sections of society.
3. India to have a centralized form of government with a strong Centre.
4. It seeks to maintain the sovereign rights on land, sea, and air according to the law of civilized nations.
Select the correct answer using the code given below:
a) 1 and 2 only
b) 2 and 3 only
c) 2,3 and 4 only
d) 1, 2, and 4 only

Ans) d
Exp) Option d is the correct answer.
Objectives Resolution was introduced by Jawahar Lal Nehru on 13th December 1946 in the Constituent
Assembly. It not only outlined the goals or objectives of the Constituent Assembly but also provided the
philosophy and guiding principles for the framing of the constitution. This Resolution was unanimously
adopted by the Constituent Assembly on 22 January 1947 as the Preamble to the Indian Constitution.
Statement 1 is correct: The Objective Resolution forms the basis of the Preamble to the Indian Constitution.
It declares the goals of the Indian Constitution, i.e. to foster unity and ensure economic and political
security. It lays down the fundamentals and philosophy of the Indian Constitution.
Statement 2 is correct: All people of India shall be guaranteed and secured social, economic, and political
justice; equality of status and opportunities and equality before the law; and fundamental freedoms – of
speech, expression, belief, faith, worship, vocation, association and action – subject to law and public
morality. To provide adequate safeguards to the minorities, backward and tribal areas, depressed, and other
backward classes.
Statement 3 is incorrect: All powers and authority of sovereign and independent India and its constitution
shall flow from the people. India to have a Federal form of government with a Division of Powers between
the Centre and the States.
Statement 4 is correct: To maintain the integrity of the territory of the Republic and the sovereign rights
on land, sea, and air according to the law of civilized nations. To attain a rightful and honored place in the
world and make a willing contribution to the promotion of world peace and the welfare of mankind.
Source: https://ncert.nic.in/ncerts/l/lehs305.pdf

Q.38) The establishment of British territorial control over India brought changes in different spheres of life.
Education was one of such areas where lot of changes came with the transfer of power to the British. The
British Government in India established many commissions in the field of education. Which of the
Commissions/committee mentioned below are related to the development of Education?
1. Hunter Commission (1882-83)
2. Aitchison Committee (1886)
3. Saddler Commission (1917-19)
4. Hartog Committee (1929)

Forum Learning Centre: Delhi - 2nd Floor, IAPL House, 19 Pusa Road, Karol Bagh, New Delhi - 110005 | Patna - 2nd floor, AG Palace, E Boring Canal Road,
Patna, Bihar 800001 | Hyderabad - 1st & 2nd Floor, SM Plaza, RTC X Rd, Indira Park Road, Jawahar Nagar, Hyderabad, Telangana 500020
9821711605 | https://academy.forumias.com | admissions@forumias.academy | helpdesk@forumias.academy
Page 26 of 33

SFG 2023 | LEVEL 1 | Test #22 – Solutions |


5. Linlithgow Commission (1926)
Select the correct answer using the code given below:
a) 1, 2, 3 and 4 only
b) 1, 3 and 4 only
c) 1, 3 and 5 only
d) 1, 2, 3, 4 and 5

Ans) b
Exp) Option b is the correct answer..
Option 1 is correct: In 1882, the Government appointed a commission under the chairmanship of W.W.
Hunter to review the progress of education in the country since the Despatch of 1854. Its
recommendations were to primary and secondary education. Primary education should be imparted
through vernacular. To transfer control of primary education to newly set up district and municipal boards.
secondary education should have two divisions literary and vocational.
Option 2 is incorrect: The Aitchison Committee on Public Services (1886), set up by Dufferin. It was
related to the civil services. It recommended for the classification of the civil service into Imperial Indian
Civil Service (examination in England), Provincial Civil Service (examination in India) and Subordinate Civil
Service (examination in India). It recommended to increase the age limit to 23.
Option 3 is correct: The. Saddler Commission (1917-19) was set up to study and report on problems of
Calcutta University, but its recommendations were applicable to other universities also. School course
should cover 12 years. Students should enter university after an intermediate stage (rather than matric) for
a three-year degree course in university. The university should function as centralised, unitary residential-
teaching autonomous body
Option 4 is correct: The Hartog Committee (1929) was set up to report on development of education. Its
main recommendations were as follows. Emphasis should be given to primary education but there need be
no hasty expansion or compulsion in education. Only deserving students should go in for high school and
intermediate stage, while average students should be diverted to vocational courses after VIII standard
Option 5 is incorrect: Linlithgow Commission (1926) inquired into the crisis of Indian agriculture. The Royal
commission of Agriculture is also known as Linlithgow commission
Knowledge Base: The British Government in India established many commission/committe in the field of
education. The commission are as follows
1) under company’s rule:
a. Charter Act of 1813
b. Lord Macaulay’s Minute (1835)
c. Wood’s Despatch (1854)
2) under British Crown:
a. Hunter Education Commission (1882-83)
b. Raleigh Commission (1902,)
c. Government Resolution on Education Policy (1913)
d. Saddler University Commission (1917-19)
e. Hartog Committee (1929)
f. Sergeant Plan of Education (1944)
Source: Ch 30, Spectrum

Q.39) With reference to the Muslim League’s Lahore Resolution of 1940, consider the following statements:
1. One of the demands was the creation of ‘Pakistan’ as a separate nation for Muslims.
2. The resolution was drafted by Sikandar Hayat Khan.

Forum Learning Centre: Delhi - 2nd Floor, IAPL House, 19 Pusa Road, Karol Bagh, New Delhi - 110005 | Patna - 2nd floor, AG Palace, E Boring Canal Road,
Patna, Bihar 800001 | Hyderabad - 1st & 2nd Floor, SM Plaza, RTC X Rd, Indira Park Road, Jawahar Nagar, Hyderabad, Telangana 500020
9821711605 | https://academy.forumias.com | admissions@forumias.academy | helpdesk@forumias.academy
Page 27 of 33

SFG 2023 | LEVEL 1 | Test #22 – Solutions |


Which of the statements given above is/are correct?
a) 1 only
b) 2 only
c) Both 1 and 2
d) Neither 1 nor 2

Ans) b
Exp) Option b is the correct answer.
Statement 1 is incorrect: The 1940 resolution of Muslim league also called as Lahore resolution never
mentioned a word Pakistan. On 23 March 1940, the League moved a resolution demanding a measure of
autonomy for the Muslim majority areas of the subcontinent. This ambiguous resolution never mentioned
partition or Pakistan.
Statement 2 is correct: Sikandar Hayat Khan, Punjab Premier and leader of the Unionist Party drafted
the Lahore resolution in 1940. He declared in a Punjab assembly speech on 1 March 1941 that he was
opposed to a Pakistan that would mean “Muslim Raj here and Hindu Raj elsewhere. He reiterated his plea
for a loose (united)confederation with considerable autonomy for the confederating units.
Source: 12th NCERT Volume 3: Understanding Partition

Q.40) Consider the following statements regarding ‘Ayushman Bharat Digital Mission (ABDM)’:
1. The participation in the Mission is mandatory for the citizens availing subsidies granted by Central
government.
2. Under the mission, patients will have opportunity to access their medical records on their mobile phone.
3. The medical records of the patients under ABDM are stored up to a maximum period of one year only.
Which of the statements given above is/are correct?
a) 1 only
b) 2 only
c) 2 and 3 only
d) 1, 2 and 3

Ans) b
Exp) Option b is the correct answer.
Ayushman Bharat Digital Mission aims to provide digital health IDs for all Indian citizens to help hospitals,
insurance firms and citizens to access health records electronically when required.
Statement 1 is incorrect: Participation in Ayushman Bharat Digital Mission is voluntary for all citizens.
Participation of a healthcare facility or an institution is also voluntary and shall be taken by the respective
management (government or private management). There is no such provision under the scheme to make
it mandatory for the citizens who are availing subsidies by the government.
Statement 2 is correct: Ayushman Bharat Digital Mission (ABDM) envisages improving the access,
efficiency, effectiveness, and transparency of the healthcare delivery system in the country. Patients will
have opportunity to link, securely store and access their medical records on their phone or online. These
may include prescriptions, diagnostic reports, discharge summaries, etc.
Statement 3 is incorrect: Ayushman Bharat Digital Mission does not store any medical records. These are
always created and stored by healthcare providers as per their retention policies and this will continue.
ABDM only facilitates secure data exchange between the intended stakeholders on ABDM network after
the patient’s consent.
Only the data collected for registries such as Health ID registry, Healthcare Professional Registry and
Healthcare Facility Registry is stored centrally. It is necessary for these datasets to be stored centrally

Forum Learning Centre: Delhi - 2nd Floor, IAPL House, 19 Pusa Road, Karol Bagh, New Delhi - 110005 | Patna - 2nd floor, AG Palace, E Boring Canal Road,
Patna, Bihar 800001 | Hyderabad - 1st & 2nd Floor, SM Plaza, RTC X Rd, Indira Park Road, Jawahar Nagar, Hyderabad, Telangana 500020
9821711605 | https://academy.forumias.com | admissions@forumias.academy | helpdesk@forumias.academy
Page 28 of 33

SFG 2023 | LEVEL 1 | Test #22 – Solutions |


because they are essential to provide interoperability, trust, and identification and single source of truth
across different digital health systems.
Knowledge Base:
Key Features of Ayushman Bharat Digital Mission:
1) The digital platform will be launched with four key features — health ID, personal health records, Digi
Doctor and health facility registry.
2) It is implemented by the National Health Authority (NHA) under the Ministry of Health and Family
Welfare.
3) The Digi Doctor option will allow doctors from across the country to enrol and their details, including
their contact numbers if they want to provide them, will be available.
4) The national health ID will be a repository of all health-related information of every Indian.
5) Every patient who wishes to have their health records available digitally must create a unique Health
ID, using their basic details and mobile or Aadhaar number.
6) Health ID will be voluntary and applicable across states, hospitals, diagnostic laboratories and
pharmacies.
Source: Ayushman Bharat Digital Health Mission - Explained, pointwise -ForumIAS Blog
NHA | Official website Ayushman Bharat Digital Mission (abdm.gov.in)

Q.41) With reference to the Cabinet Mission, which of the following statements is/are correct?
1. It recommended a federal government.
2. It enlarged the powers of the Indian courts.
3. It provided for more Indians in the ICS.
Select the correct answer using the code given below.
a) 1 only
b) 2 and 3
c) 1 and 3
d) None

Ans) a
Exp) Option a is the correct answer.
Option 1 is correct: The Cabinet Mission recommended a loose three-tier confederation. A united India
was to have a weak central government controlling only foreign affairs, defence and communications with
the existing provincial assemblies being grouped into three sections while electing the constituent
assembly. Thus, Cabinet Mission plan proposed a weak Centre with provincial autonomy- essentially
proposing a federal structure of government.
Option 2 and 3 are incorrect: There was no provision with respect to enlarging the powers of Indian
courts and increasing strength of Indians in ICS.
Source: UPSC CSE Pre 2015

Q.42) With reference to the Unionist party, consider the following statements:
1. Sir Syed Ahmed Khan was one of the prominent members of this party.
2. The party mainly defended the rights of landless labourers in Punjab against the landlords.
3. The party supported Muslim League’s demand for partition of India.
Which of the statements given above is/are correct?
a) 1 and 3 only
b) 3 only
c) 2 only
d) None of the above

Forum Learning Centre: Delhi - 2nd Floor, IAPL House, 19 Pusa Road, Karol Bagh, New Delhi - 110005 | Patna - 2nd floor, AG Palace, E Boring Canal Road,
Patna, Bihar 800001 | Hyderabad - 1st & 2nd Floor, SM Plaza, RTC X Rd, Indira Park Road, Jawahar Nagar, Hyderabad, Telangana 500020
9821711605 | https://academy.forumias.com | admissions@forumias.academy | helpdesk@forumias.academy
Page 29 of 33

SFG 2023 | LEVEL 1 | Test #22 – Solutions |


Ans) d
Exp) Option d is the correct answer.
Statement 1 is incorrect: Sir Syed Ahmed Khan is not a member of the Unionist party. Sir Sikandar Hayat
Khan, Sir Fazli Husain, Sir Shahab-ud-Din, Muhammad Hussain Shah and Sir Chhotu Ram were all
members of the party. Although a majority of Unionists were Muslims, a large number of Hindus and Sikhs
also supported and participated in the Unionist Party.
Statement 2 is incorrect: Unionists party mainly represented the interest of landlords (and not the
landless labourers). It was a political party representing landholders- Sikhs, Muslims and Hindus in the
Punjab.
Statement 3 is incorrect: Unionist Party leader Malik Khizr Hayat Khan Tiwana played a key role in limiting
the Muslim League’s influence in Punjab Province during 1942 to 1947. He strongly opposed the Muslim
League’s demand for a separate Pakistan and presented his own vision of united Punjab within a
decentralised Federal India.
Source: 12th NCERT volume 3: Understanding Partition
Book: Khizr Tiwana, the Punjab Unionist Party and the Partition of India

Q.43) With respect to the history of demand for creation of Pakistan as a separate nation, consider the
following statements
1. Choudhry Rahmat Ali presented a pamphlet to the British during a round table conference, demanding a
separate nation of Pakistan.
2. The demand of Pakistan as a separate nation have been traced back to the Urdu poet Mohammad Iqbal.
3. The Launch of direct-action day by Muhamad Ali Jinnah accentuated the demand for Pakistan as a
separate nation.
Which of the statements given above are correct?
a) 1 and 2 only
b) 2 and 3 only
c) 1 and 3 only
d) 1, 2 and 3

Ans) c
Exp) Option c is the correct answer.
Statement 1 is correct: Choudhry Rehmat Ali issued a pamphlet to the British and Indian delegates in the
Third Round Table Conference in London. But these were dismissed as students' ideas. Choudhry Rehmat
Ali, a law student at the University of Cambridge in 1933, designed a pamphlet "Now or Never; Are We to
Live or Perish Forever?", also known as the Pakistan Declaration. The pamphlet talked about providing
national status to Pakistan-- “It embodies their demand for the recognition of their national status, as
distinct from the other inhabitants of India, by the grant to Pakistan of a separate Federal Constitution
on religious, social and historical grounds.”
Statement 2 is incorrect: Mohammad Iqbal called for reorganisation of Muslim-majority areas in north-
western India into an autonomous unit within a single, loosely structured Indian Federation but didn't
visualise the emergence of a separate country. So, the demand of Pakistan as a separate nation cannot
be traced back to the Urdu poet Mohammad Iqbal.
Statement 3 is correct: Muhamad Ali Jinnah launched direct action day in Calcutta on August 16, 1946,
with a slogan “We will fight and get Pakistan”. It gave a fillip to the demand for creation of Pakistan as a
separate nation.
Source: Spectrum: The Brief History of Modern India

Forum Learning Centre: Delhi - 2nd Floor, IAPL House, 19 Pusa Road, Karol Bagh, New Delhi - 110005 | Patna - 2nd floor, AG Palace, E Boring Canal Road,
Patna, Bihar 800001 | Hyderabad - 1st & 2nd Floor, SM Plaza, RTC X Rd, Indira Park Road, Jawahar Nagar, Hyderabad, Telangana 500020
9821711605 | https://academy.forumias.com | admissions@forumias.academy | helpdesk@forumias.academy
Page 30 of 33

SFG 2023 | LEVEL 1 | Test #22 – Solutions |


Q.44) With reference to the freedom fighter Subhash Chandra Bose, consider the following statements:
1. He established the Forward bloc party in 1939 within the congress.
2. He was elected as president of All India Kisan Sabha in 1930.
3. He resigned from Civil service in response to the Non-Cooperation movement.
Which of the statements given above is/are correct?
a) 1 and 2 only
b) 1 and 3 only
c) 3 only
d) 1, 2 and 3

Ans) b
Exp) Option b is the correct answer.
Statement 1 is correct: Subhash Bose was elected as president of Tripuri Congress session in 1939 after
defeating the Gandhiji’s candidate Pattabi Sitharamaya. Following the worsening relationship between
Subhash Chandra Bose and moderates in Congress, he and his followers formed the Forward Bloc in 1939
as a new party within the Congress. The purpose was to consolidate the political left and major support
base in his home state Bengal.
Statement 2 is incorrect: In 1930 he was elected Mayor of Calcutta, the same year he was elected
President of All India Trade Union Congress (not All India Kisan Sabha). In fact, Subhash Chandra Bose
had never been a president of All India Kisan Sabha.
Statement 3 is correct: He resigned from a prestigious post in the Indian Civil Service in response to
the launching of the Non-Cooperation movement in 1922. In the late 1920s, he was amongst the first
Congress leaders to call for complete independence from Britain (Purna Swaraj), rather than the previous
Congress objective of India becoming a British dominion.
Source: India’s Struggle for Independence: Bipin Chandra

Q.45) Which among the following statements is incorrect about the liquefaction of soil during earthquakes?
a) Liquefaction potential of sand can affect the collapse of structures during earthquakes.
b) Irregular Grain Shape of the sand increases the soil liquification.
c) Glass beads-shaped sand with higher roundness and sphericity are liquefied first during earthquakes.
d) Manufactured sand provides stability against liquefaction.

Ans) b
Exp) Option b is the correct answer.
Liquefaction of sand is a phenomenon in which the strength and stiffness of soil is reduced by earthquake
shaking or other rapid loading and lead to the collapse of structures resting on the liquefied ground.
Statement a is correct: There is a strong relationship between the grain shape of sands and their
liquefaction potential. The liquefaction potential of sand is one of the major factors behind the collapse
of structures during earthquakes.
Statements c and d are correct: The glass bead-shaped sand, which has a regular shape with higher
roundness and sphericity, liquefied first.
The river sand, whose roundness and sphericity fall between glass beads and manufactured sand, liquefied
next, followed by manufactured sand, whose shape is relatively irregular.
As the irregular shape increase the stability and hence provide stability against liquefaction.
Statement b is incorrect: Irregular grain shape of the sand increases soil stability and hence the less
liquification (Hence option b is incorrect). This is because the shear force required to break the inter-
particle locking is more for the grains with relatively irregular shapes. As the shape of the particles

Forum Learning Centre: Delhi - 2nd Floor, IAPL House, 19 Pusa Road, Karol Bagh, New Delhi - 110005 | Patna - 2nd floor, AG Palace, E Boring Canal Road,
Patna, Bihar 800001 | Hyderabad - 1st & 2nd Floor, SM Plaza, RTC X Rd, Indira Park Road, Jawahar Nagar, Hyderabad, Telangana 500020
9821711605 | https://academy.forumias.com | admissions@forumias.academy | helpdesk@forumias.academy
Page 31 of 33

SFG 2023 | LEVEL 1 | Test #22 – Solutions |


becomes irregular, they get interlocked with each other during shearing. Interlocking provides
additional resistance to shear, and hence the tendency to get separated from each other to float in the
fluid becomes lesser for particles with irregular shapes.
Source: https://pib.gov.in/PressReleaseIframePage.aspx?PRID=1842680#:

Q.46) Which of the following statements is/are incorrect features of the elections of 1946 in British India?
1. The election campaign of the Congress was strongly anti-Muslim League in character.
2. It was the first election in British India fought on the basis of universal adult franchise.
Select the correct answer using the code given below:
a) 1 only
b) 2 only
c) Both 1 and 2
d) Neither 1 nor 2

Ans) c
Exp) Option c is the correct answer.
In July 1945, Labour Party formed the government in Britain. Clement Attlee took over as the new Prime
Minister and Pethick Lawrence as the new secretary of state for India. In August 1945, elections to central
and provincial assemblies were announced.
Statement 1 is incorrect: The election campaign of the Congress was anti-British in its thrust. Congress
slogans were “Release the misguided patriots” of the Indian National Army and “punish the guilty” officials
who committed excesses in 1942. It was not anti-League in its tenor despite the elections being fought
by the League on the issue of Pakistan.
Statement 2 is incorrect: The franchise was very limited as for the provinces, less than 10 per cent of the
population could vote and for the Central Assembly, less than 1 per cent of the population was eligible.
Source: Spectrum Modern India 2019-20 Edition

Q.47) Consider the following pairs regarding the Newspapers/ Journals and their Founders/ Editors:
Newspapers/ Founders/
Journals Editors
1. Indian Mirror Devendranath
Tagore
2. Sudharak Gopal Ganesh
Agarkar
3. Voice of India Dadabhai
Naoroji
Which of the pairs given above are correct?
a) 1 and 2 only
b) 1 and 3 only
c) 2 and 3 only
d) 1, 2 and 3

Ans) d
Exp) Option d is the correct answer.
Although, the British were responsible for bringing the printing press in India, they were most allergic to
the emergence of a newspaper in this country. Some of the Newspapers/ Journals and their founders are
given below.

Forum Learning Centre: Delhi - 2nd Floor, IAPL House, 19 Pusa Road, Karol Bagh, New Delhi - 110005 | Patna - 2nd floor, AG Palace, E Boring Canal Road,
Patna, Bihar 800001 | Hyderabad - 1st & 2nd Floor, SM Plaza, RTC X Rd, Indira Park Road, Jawahar Nagar, Hyderabad, Telangana 500020
9821711605 | https://academy.forumias.com | admissions@forumias.academy | helpdesk@forumias.academy
Page 32 of 33

SFG 2023 | LEVEL 1 | Test #22 – Solutions |


Pair 1 is correct: Indian Mirror is the first Indian daily paper in English. It was started by Devendranath
Tagore. In October 1839, he along with his friends started the Tattwaranjini Sabha which was later renamed
to Tattwabodhini Sabha. He was also a founder of the Brahmo religion in 1848.
Pair 2 is correct: Gopal Ganesh Agarkar started his own periodical, Sudharak, which spoke against
untouchability and the caste system. He was a cofounder of the New English School, the Deccan Education
Society and Fergusson College. He was a principal of Fergusson College. He was also the first editor of
Kesari, the journal started by Lokmanya Tilak.
Pair 3 is correct: Voice of India was started by Dadabhai Naoroji. In 1867, he founded the East India
Association in London. In 1874, he was appointed the Dewan of Baroda and in 1875 he was elected a Member
of the Municipal Corporation, Bombay. He is Known as ‘The Grand Old Man of India’.
Source: https://www.inc.in/leadership/past-party-presidents/shri-dadabhai-naoroji
Pg 216, ch 9, Spectrum

Q.48) Consider the following statements with respect to Gandhi's attitude towards actions of Indian
National Army (INA):
1. Gandhiji condemned Indian National Army’s action of taking help from Nazi Germany, to liberate India
from British rule.
2. Gandhi praised the Indian National Army for its secular nature.
3. Gandhi supported the cause of freedom of Indian National Army’s prisoners from British.
Which of the statements given above are correct?
a) 1 and 2 only
b) 2 and 3 only
c) 1 and 3 only
d) 1, 2 and 3

Ans) b
Exp) Option b is the correct answer.
Statement 1 is incorrect: Gandhi did not condemn the Indian National Army’s action of seeking help
from Nazi Germany. His conversation with American Journalist Louis Fischer shows Gandhi’s contempt
for British rule in India. Gandhiji observed “There are powerful elements of Fascism in British rule”. So
according to him there was nothing wrong in taking the help of Nazi Germany in Fight against the
British rule.
Statement 2 and 3 are correct: Gandhi has a difference of opinion with the violent way of working of Indian
National Army. Despite this he praised the INA for its secular nature and bringing together the Indian
soldiers without discriminating on the basis of religion or caste. He also admired the selfless sacrifice
of INA soldiers and supported the cause of their freedom from British during Redfort Trials.
Source: India’s Struggle for Independence: The Quit India movement and the INA
http://magazines.odisha.gov.in/Orissareview/2019/Octo/engpdf/56-64.pdf (pg no 58)

Q.49) With reference to the role of Subhas Chandra Bose in Freedom Struggle of India, consider the
following statements
1. He declared war against both Britain and the United States of America (USA).
2. He secured a promise from the Japanese government that Japan had no intention of occupying India.
3. He set up the headquarter of Indian National Army in Andaman and Nicobar Islands.

Forum Learning Centre: Delhi - 2nd Floor, IAPL House, 19 Pusa Road, Karol Bagh, New Delhi - 110005 | Patna - 2nd floor, AG Palace, E Boring Canal Road,
Patna, Bihar 800001 | Hyderabad - 1st & 2nd Floor, SM Plaza, RTC X Rd, Indira Park Road, Jawahar Nagar, Hyderabad, Telangana 500020
9821711605 | https://academy.forumias.com | admissions@forumias.academy | helpdesk@forumias.academy
Page 33 of 33

SFG 2023 | LEVEL 1 | Test #22 – Solutions |


Which of the statements given above are correct?
a) 1 and 2 only
b) 2 and 3 only
c) 1 and 3 only
d) 1, 2 and 3

Ans) a
Exp) Option a is the correct answer.
Statement 1 is correct: The provisional government established by Subhas Chandra Bose declared war on
Britain and the USA and was recognised by the Axis powers and their satellites.
Statement 2 is correct: Subhash Chandra Bose went to Tokyo in 1943 and met Japanese PM Tojo. Tojo
declared that Japan had no intention of occupying Indian territory.
Statement 3 is incorrect: Subhash Bose after reorganising the Indian National Army set up two
headquarters, in Rangoon and in Singapore (not in Andaman and Nicobar Islands).
Source: India’s Struggle for Independence: Quit India movement and the INA

Q.50) Consider the following statements regarding Tiger Conservation:


1. Tiger has been classified as “Endangered” under IUCN Red List.
2. Government of India has launched Project Tiger in 1973, to comply with St. Petersburg Declaration on
Tiger Conservation.
3. India achieved the goal of doubling the tiger numbers in 2018, four years in advance from the targeted
year 2022.
Which of the statements given above are correct?
a) 1 and 2 only
b) 2 and 3 only
c) 1 and 3 only
d) 1, 2 and 3

Ans) c
Exp) Option c is the correct answer.
Recently, India has hosted the Pre-Summit meeting of Tiger Range Countries (TRCs).
Statement 1 is correct: Panthera Tigris i.e., Tiger is categorized as Endangered as per the IUCN red data
book.
Statement 2 is incorrect: St. Petersburg Declaration on Tiger Conservation was an outcome of St.
Petersburg Tiger Summit, 2010. As per this declaration the 13 tiger range countries pledged to achieve a
target of doubling the number of Tigers. As, Project Tiger was launched in 1973, it was not a consequence
of St. Petersburg Tiger Summit.
Statement 3 is correct: A per the data released by the MoEF&CC:
1) India is home to 53 Tiger Reserves covering approximately 75,000 Sq. Km area in 18 States with
approximately 75% population of the wild tiger at global level.
2) India achieved the goal of doubling the tiger numbers in 2018 itself, four years in advance from the
targeted year 2022.
3) Also, so far 17 Tiger Reserves in the country have got CA|TS international accreditation and two Tiger
Reserves have got International Tx2 Award.
Source: https://www.worldwildlife.org/species/tiger
https://pib.gov.in/PressReleasePage.aspx?PRID=1850581#:~:text=Pre%2Dsummit%20meeting%20of%20
the%20Tiger%20Range%20Countries%20as%20a,in%20progress%20in%20New%20Delhi.

Forum Learning Centre: Delhi - 2nd Floor, IAPL House, 19 Pusa Road, Karol Bagh, New Delhi - 110005 | Patna - 2nd floor, AG Palace, E Boring Canal Road,
Patna, Bihar 800001 | Hyderabad - 1st & 2nd Floor, SM Plaza, RTC X Rd, Indira Park Road, Jawahar Nagar, Hyderabad, Telangana 500020
9821711605 | https://academy.forumias.com | admissions@forumias.academy | helpdesk@forumias.academy

You might also like